McGraw-Hill Education ACT 2017 (2016)

Part IV. THREE PRACTICE TESTS

ACT PRACTICE TEST 2

Answer Sheet

ENGLISH

Image

Image

Image

Image

Image

Image

Image

Image

Image

10 Image

11 Image

12 Image

13 Image

14 Image

15 Image

16 Image

17 Image

18 Image

19 Image

20 Image

21 Image

22 Image

23 Image

24 Image

25 Image

26 Image

27 Image

28 Image

29 Image

30 Image

31 Image

32 Image

33 Image

34 Image

35 Image

36 Image

37 Image

38 Image

39 Image

40 Image

41 Image

42 Image

43 Image

44 Image

45 Image

46 Image

47 Image

48 Image

49 Image

50 Image

51 Image

52 Image

53 Image

54 Image

55 Image

56 Image

57 Image

58 Image

59 Image

60 Image

61 Image

62 Image

63 Image

64 Image

65 Image

66 Image

67 Image

68 Image

69 Image

70 Image

71 Image

72 Image

73 Image

74 Image

75 Image

MATHEMATICS

Image

Image

Image

Image

Image

Image

Image

Image

Image

10 Image

11 Image

12 Image

13 Image

14 Image

15 Image

16 Image

17 Image

18 Image

19 Image

20 Image

21 Image

22 Image

23 Image

24 Image

25 Image

26 Image

27 Image

28 Image

29 Image

30 Image

31 Image

32 Image

33 Image

34 Image

35 Image

36 Image

37 Image

38 Image

39 Image

40 Image

41 Image

42 Image

43 Image

44 Image

45 Image

46 Image

47 Image

48 Image

49 Image

50 Image

51 Image

52 Image

53 Image

54 Image

55 Image

56 Image

57 Image

58 Image

59 Image

60 Image

READING

Image

Image

Image

Image

Image

Image

Image

Image

Image

10 Image

11 Image

12 Image

13 Image

14 Image

15 Image

16 Image

17 Image

18 Image

19 Image

20 Image

21 Image

22 Image

23 Image

24 Image

25 Image

26 Image

27 Image

28 Image

29 Image

30 Image

31 Image

32 Image

33 Image

34 Image

35 Image

36 Image

37 Image

38 Image

39 Image

40 Image

SCIENCE

Image

Image

Image

Image

Image

Image

Image

Image

Image

10 Image

11 Image

12 Image

13 Image

14 Image

15 Image

16 Image

17 Image

18 Image

19 Image

20 Image

21 Image

22 Image

23 Image

24 Image

25 Image

26 Image

27 Image

28 Image

29 Image

30 Image

31 Image

32 Image

33 Image

34 Image

35 Image

36 Image

37 Image

38 Image

39 Image

40 Image

You may wish to remove these sample answer document pages to respond to the practice ACT Writing Test.

Image

ENGLISH TEST

45 Minutes – 75 Questions

DIRECTIONS: In the passages that follow, some words and phrases are underlined and numbered. In the answer column, you will find alternatives for the words and phrases that are underlined. Choose the alternative that you think is best and fill in the corresponding bubble on your answer sheet. If you think that the original version is best, choose “NO CHANGE,” which will always be either answer choice A or F. You will also find questions about a particular section of the passage, or about the entire passage. These questions will be identified by either an underlined portion or by a number in a box. Look for the answer that clearly expresses the idea, is consistent with the style and tone of the passage, and makes the correct use of standard written English. Read the passage through once before answering the questions. For some questions, you should read beyond the indicated portion before you answer.

PASSAGE I

The following paragraphs may or may not be in the most logical order. You may be asked questions about the logical order of the paragraphs, as well as where to place sentences logically within any given paragraph.

Noh Theater

[1]

Noh is a highly ritualized form of drama that Image Medieval Japan as a type of play performed in front of nobility. Noh theater reached its apex in the fourteenth and fifteenth centuries with the works of a Image and it Image

1.   A.  NO CHANGE

B.  original to

C.  originating in

D.  originated in

2.   F.  NO CHANGE

G.  playwright, named Kannami, and his son, Zeami,

H.  playwright named Kannami and his son Zeami;

 J.  playwright named Kannami; and his son Zeami,

3.   A.  NO CHANGE

B.  has remained largely unchanged.

C.  will be largely unchanged.

D.  will largely remain unchanged.

[2]

There are certain traits that make Noh unique in the Japanese theatrical world. The stage is always sparse, Image with a painting of a pine tree as a backdrop. Props are minimal and often symbolic. Image and it usually symbolizes another object. The costumes are lavish and colorful, and the colors of the costumes are also symbolic. There is a chorus that often narrates, along with instrumentalists who add to the ambience with the unique and otherworldly scores Image

4.   F.  NO CHANGE

G.  decorated solely

H.  just decorated solely

 J.  decorated only solely

5.   A.  NO CHANGE

B.  The fan, for example, is a staple of Noh theater,

C.  The fan for example, is a staple of Noh theater,

D.  The fan, for example is a staple, of Noh theater,

6.   F.  NO CHANGE

G.  they play for

H.  they play

 J.  it will play

[3]

Image If the audience is familiar with Noh, it can recognize the characters in Image the stylized masks that the actors wear. Certain masks represent certain types of characters and are intended to show specific traits possessed by these characters. The masks are intentionally painted in such a way that the different angles actually look like different facial expressions. Image

7.   Which of the following sentences (assuming all are true) if added here, would best introduce the new subject of Paragraph 3?

A.  In the early days, Noh theater was sponsored by the elite rulers of Japan.

B.  Japanese theater has been popular for centuries.

C.  Masks play an important role in Noh theater.

D.  There are archetypal characters who show up repeatedly in the repertoire of plays.

8.   F.  NO CHANGE

G.  with

H.  by

 J.  for

9.   At this point, the writer would like to highlight a very special talent that Noh actors must develop in order to be convincing. Which of the following sentences (assuming all are true) if added here, would most successfully achieve this effect?

A.  The actors wearing them must be skilled at tilting their heads in order to express nuances in emotion.

B.  The masks, the actors wear, are colorful and detailed and truly works of art.

C.  The actors must learn to express themselves in ways that are often unfamiliar to viewers of Western theater.

D.  Noh actors begin training at a very young age, so by the time they are much older, they have become very accomplished in their trade.

[4]

Image It is a very sophisticated and subtle form of drama, and according to legend, possesses something called yugen. An approximate English translation of this Image concept refers to mystery and to what lies beneath the surface.

10.   F.  NO CHANGE

G.  Noh theater combines poetry dance and, music, and often deals with supernatural themes.

H.  Noh theater combines poetry, dance and music—and often deals with supernatural themes.

 J.  Noh theater combines poetry, dance, and music, and often deals with supernatural themes.

11.   Which of the following alternatives for the underlined portion would be LEAST acceptable?

A.  complex

B.  theoretical

C.  representational

D.  summarized

[5]

[1] Most of the plays being performed today are the originals written by Kannami and Zeami, although a few new ones Image since then. [2] Noh is not the most popular form of theater in Japan today, Imageperformers are extremely dedicated, and people still buy tickets to enjoy this classic art form. [3] The fact that it has remained essentially in its original form for over 600 years Image to its incredible beauty, mystique, and lasting elegance.

12.   F.  NO CHANGE

G.  will have been written

H.  have been wrote

 J.  have been written

13.   A.  NO CHANGE

B.  also its

C.  but its

D.  because it’s

14.   F.  NO CHANGE

G.  will speak

H.  speaks

 J.  speaked

Question 15 asks about the preceding passage as a whole.

15.   In reviewing notes, the writer discovers that the following information has been left out of the essay:

Zeami also wrote a treatise on the methodology of Noh, which is still studied by Noh actors.

If added to the essay, the sentence would most logically be placed after Sentence:

A.  2 in Paragraph 2.

B.  1 in Paragraph 5.

C.  2 in Paragraph 3.

D.  3 in Paragraph 5.

PASSAGE II

Calligraphy: Beautiful Writing

[1]

Art takes many forms, including watercolor painting, pencil sketching, Image One lesser known and perhaps less appreciated art form is calligraphy, the elegant script of letters and figures. Many modern-day computer fonts are attempts to replicate this ancient art. Image

16.   F.  NO CHANGE

G.  photography, sculpture.

H.  photography and to sculpture.

 J.  photography, and sculpture.

17.   The writer is considering adding the following true statement after the preceding sentence:

Computer fonts, however, cannot fully replicate the artistry and talent of an accomplished calligrapher.

Would this be a relevant addition to the paragraph?

A.  Yes, because the writer goes on to discuss how calligraphy is an art form.

B.  Yes, because the passage continues to make references to modern technology.

C.  No, because the writer is focusing on calligraphy itself, not on specific calligraphers.

D.  No, because computer fonts have nothing to do with the art of calligraphy.

The word calligraphy is Image the Greek words kalli, which means “beautiful,” and graphia, which means “writing.” Image as many ancient peoples relied upon the written word and had some form of written records. Since the printing press wasn’t invented until the mid-fifteenth century, legible handwriting was an important and useful skill Image the known world.

18.   F.  NO CHANGE

G.  derived with

H.  derived by

 J.  derived to

19.   A.  NO CHANGE

B.  It is with difficulty that it is said which civilization calligraphy emerges from,

C.  From which civilization calligraphy directly emerged is difficult to say,

D.  Which civilization, it is difficult to say, from which calligraphy directly emerged,

20.   F.  NO CHANGE

G.  all around

H.  throughout which

 J.  around which

Chinese calligraphy Image nearly 5,000 years. Around 200 B.C., a 3,000-character index was established Image Chinese scholars. These scribes Image their own styles Image by varying the thickness of the lines, the amount of ink, and the types of paper. Image true “artists of script” emerged, and the Image Chinese calligraphy around the seventh century, developing their own style, which included an appreciation Image imperfection as well as technical ability.

21.   A.  NO CHANGE

B.  dated back to

C.  dates back

D.  dated back

22.   F.  NO CHANGE

G.  for the use by

H.  for the use with

 J.  for use by

23.   A.  NO CHANGE

B.  quickly developing

C.  quickly developed

D.  who have quickly developed

24.   F.  NO CHANGE

G.  when replicating the various characters

H.  when replicating

 J.  OMIT the underlined portion

25.   A.  NO CHANGE

B.  Soon,

C.  Yet,

D.  Otherwise,

26.   F.  NO CHANGE

G.  Japanese, adapted

H.  Japanese adapted,

 J.  Japanese having adapted

27.   A.  NO CHANGE

B.  with

C.  to

D.  unto

In Europe, Image during the Middle Ages. Manual recording and duplication of religious texts demanded an abundance of beautiful handwriting. A variety of styles soon emerged, including Gothic calligraphy. In the Gothic style, letters are Image and lines are much narrower than in other styles. Because the print takes up less space, less paper is required.

28.   F.  NO CHANGE

G.  the development of calligraphy greatly influenced by the Church

H.  the Church greatly influenced the development of calligraphy

 J.  calligraphy greatly influenced by the development of the Church

29.   A.  NO CHANGE

B.  closely

C.  more closely spaced

D.  spaced closely

Today, calligraphy continues to fascinate both scribes and art aficionados alike. Modern calligraphy equipment, such as specialized pens, inks, and paper, Image the art fairly easy to learn.

30.   F.  NO CHANGE

G.  making

H.  make

 J.  made

PASSAGE III

Early American Fur Trappers

The myth of the early American mountain men Image a picture of romance, adventure, and intrigue. In reality, most mountain men were fur traders Image in a tough business that sent them for months at a time to the vast rivers and mountains of the American West. For the most part, beaver pelts were the primary target of these unconventional businessmen, as beaver hats and coats were all the rage in early American towns and cities. Image

31.   A.  NO CHANGE

B.  paint

C.  will paint

D.  painting

32.   F.  NO CHANGE

G.  participating

H.  who, acting to participate

 J.  choosing to act and participate

33.   At this point, the writer is considering adding the following sentence:

While not inexpensive, harvesting beaver pelts directly from North America was far cheaper than importing them from across the ocean.

Would this be a relevant addition to make here?

A.  Yes, because the writer needs to establish that beaver pelts were very expensive.

B.  Yes, because the sentence emphasizes the importance of the American mountain man’s contribution.

C.  No, because the paragraph focuses on the American mountain man, not on beaver pelts.

D.  No, because beaver pelts from other countries cost more than those obtained in America.

While some fur trappers and traders traveled alone, many worked together in groups for a particular trading company. The Hudson Bay Company, well-known throughout Europe, was the world’s Image fur-trading company. Its Image would rendezvous at designated sites in America where trappers presented furs in exchange for money or essential goods. Image While the mountain man Imagepersonify “rugged individualism,” he was completely dependent upon his ability to trap wild animals and, therefore, relied upon consumer Image for those pelts. While some of the trappers were Image a particular fur company, others chose to be freelancers. Men Image were called “engagers,” and all furs they obtained were company property Image The “free-trapper” was the most autonomous of Image trapped wherever and with whomever he chose. He also traded or sold his furs at his own discretion. Although the free-trappers were considered by their peers to be tough and hardy Image their ability to endure the hardships of mountain living, many of these mountain men eventually Image to those hardships.

34.   F.  NO CHANGE

G.  first and, largest

H.  first and largest;

 J.  first, and, largest

35.   A.  NO CHANGE

B.  product buyers

C.  buyers of products

D.  buyers

36.   Given that all of the following sentences are true, which one should be placed here to offer a logical explanation for why trappers sometimes traded their furs for goods instead of money?

F.  While mountain men were skilled hunters and could capture their own food, they still needed many supplies in order to survive.

G.  Many Indian tribes were willing to trade goods and supplies with the mountain men.

H.  Some mountain men had families back in the cities and towns, so money was important.

 J.  Trappers enjoyed trading goods and supplies among themselves, as long as the Hudson Bay Company approved.

37.   A.  NO CHANGE

B.  appeared to

C.  appear

D.  appears to

38.   F.  NO CHANGE

G.  their demand

H.  demanded

 J.  demand

39.   A.  NO CHANGE

B.  under the employment of

C.  employed by

D.  DELETE the underlined portion.

40.   F.  NO CHANGE

G.  hired directly, by a fur company

H.  hired directly by a fur company

 J.  hired, directly by a fur company

41.   A.  NO CHANGE

B.  that were given directly to the company

C.  and did not belong to them

D.  DELETE the underlined portion.

42.   F.  NO CHANGE

G.  all:

H.  all, he

 J.  all he

43.   A.  NO CHANGE

B.  in regards with

C.  regards to

D.  irregardless of

44.   F.  NO CHANGE

G.  succumbing

H.  will be succumbing

 J.  succumbed

Question 45 asks about the preceding passage as a whole.

45.   Suppose the writer had intended to write an essay that explored the myth of the American mountain man. Would this essay successfully fulfill the writer’s goal?

A.  No, because the essay focuses on American myths in general, not just the myth of the American mountain man.

B.  No, because American mountain men did not actually exist.

C.  Yes, because the writer explains how the American mountain man story is really a myth.

D.  Yes, because the writer discusses the contrast between the romantic, mythical side of the mountain man’s life and the reality of his job.

PASSAGE IV

The Green Bay Packers

In 1919, Curly Lambeau returned home to Green Bay, Wisconsin Image In a conversation with his friend George Calhoun, he expressed regret at not being able to play football since returning home. Calhoun Image that Curly start a team in his home town. Excited by the idea, Lambeau convinced his boss at the Indian Packing Company to donate uniforms and the use of an athletic field.

46.   F.  NO CHANGE

G.  from playing football at Notre Dame, due to a severe case of tonsillitis.

H.  from a case of severe tonsillitis, which was due to playing football at Notre Dame.

 J.  from playing football at Notre Dame, which was due to a severe case of tonsillitis.

47.   A.  NO CHANGE

B.  recommended to him a decision

C.  recommended

D.  gives his recommendation

Curly ran ads in the local newspaper, inviting other athletes to join the new team. Only 20 football players joined the team the first year. Although Lambeau named the team the Big Bay Blues, Image

48.   F.  NO CHANGE

G.  the Packers called the fans and players the team.

H.  the team was called the Packers by the fans and the players.

 J.  the fans called the team the Packers, the players, too.

The conditions under which the Packers played during that first year were a far cry from those enjoyed by modern Image football teams. They played their games in an empty field behind Hagemeister Brewery. There were no locker rooms, so players normally changed Image There were no Image or accurately count attendance. Without fences and stands, the only way Image money was Image a hat around to spectators for donations. Image

49.   A.  NO CHANGE

B.  contemporary

C.  up-to-date

D.  DELETE the underlined portion

50.   F.  NO CHANGE

G.  before the game into their uniforms at home.

H.  uniforms at their home before the game.

 J.  into their uniforms, which were at home before the game.

51.   A.  NO CHANGE

B.  gates or bleachers, so there was no way to charge admission

C.  gates, or bleachers, so there was no way to charge admission

D.  gates or bleachers. So there was no way to charge admission

52.   F.  NO CHANGE

G.  it raised

H.  to raise

 J.  they could raise any

53.   A.  NO CHANGE

B.  was, quite literally to pass,

C.  was quite literally, to pass

D.  was, quite literally, to pass

54.   The writer is considering changing the first sentence of this paragraph (assuming that if there is an error, it has been fixed). Which sentence would be the best choice?

F.  The writer should not replace the sentence.

G.  The Packers endured brutal conditions in the first year, all for the love of the game.

H.  When the Packers played their first season, professional football was not very popular nationwide.

 J.  Equipped with a popular new name, the Packers were ready to begin their first season.

In 1920, bleachers were built on one side of Hagemeister Park, located behind the brewery. The largest recorded attendance at that location was 6,000 fans for the game against the Minneapolis Marines on October 23, 1921. That was the Packers’ first official game Image as part of the new American Professional Football Association, which is now known as the National Football League. Image

55.   A.  NO CHANGE

B.  they played

C.  for playing

D.  DELETE the underlined portion

56.   The writer would like to link the information already presented about the Green Bay Packers to the information in this paragraph. Assuming all are true, which of the following sentences best achieves this effect?

F.  Vince Lombardi coached the Packers with great success in the 1960s.

G.  The Packers are the only publicly owned team in the NFL.

H.  In the 1950s, Curly Lambeau was fired by the Packers as part of an internal power struggle.

 J.  This historic game marked the beginning of the Green Bay Packers, one of the oldest franchises in professional football.

From their humble beginnings, the Packers have gone on to win more NFL championships than any other team, including four Super Bowls. The Packers now play in a newly renovated stadium Image Lambeau Field Image The stadium now seats 72,515—and over 60,000 people are on the waiting list for season tickets! The team has come a long Image

57.   A.  NO CHANGE

B.  named after

C.  named

D.  naming

58.   F.  NO CHANGE

G.  after the team’s legendary founder.

H.  after the legend of the team’s founder.

 J.  after the team founder’s legendary status.

59.   A.  NO CHANGE

B.  way, from wearing donated uniforms, and passing a hat around a nearly empty field.

C.  way from wearing donated uniforms; and passing a hat around a nearly empty field.

D.  way from wearing donated uniforms and passing a hat around, a nearly empty field.

Question 60 asks about the preceding passage as a whole.

60.   Suppose the writer had been assigned to write a brief essay illustrating the economic influence of the Packers on the city of Green Bay. Would this essay fulfill that assignment?

F.  Yes, because the essay indicates that the team relied on a corporate sponsorship to get started.

G.  Yes, because the essay indicates that the team has been very successful.

H.  No, because the essay primarily focuses on how the team was started and its eventual success.

 J.  No, because the essay notes that the team relied on donations rather than charging admission.

PASSAGE V

Prepare for the Starfish Inn

“Are we really planning on staying here?” Sophie asked me incredulously. “I feel like we have no choice!” I responded. The place in question was the Starfish Inn, a motel of dubious character on the beach in Jacksonville, Florida. We ended up here Image our own irresponsibility. It was our freshman year of college, Image yearning to escape the cold and dreary weather for the sun of spring break, we decided to head south. It was a last-minute decision; we did not make reservations anywhere.

61.   A.  NO CHANGE

B.  instead of

C.  because of

D.  in part of

62.   F.  NO CHANGE

G.  and

H.  but

 J.  where

When we arrived in Florida, we tried to book a room in a decent, affordable hotel. After visiting six hotels and finding no vacancy, we Image an information booth. A kind and helpful woman delivered the discouraging news that, if we didn’t have reservations anywhere, it would be very difficult for us to Image lodging. She recommended that we check a couple of places, but they all seemed far beyond our limited ImageThen she said that the Starfish Inn was reasonably priced, but that she would not want her daughters to stay there!

63.   A.  NO CHANGE

B.  were stopped by

C.  had to stop in

D.  will stop at

64.   Which choice provides the most appropriate image?

F.  NO CHANGE

G.  secure

H.  capture

 J.  grab

65.   A.  NO CHANGE

B.  budget.

C.  budget. Our budget was pretty typical for college students.

D.  budget. I wanted to have enough money left to buy souvenirs.

So Image how we got into our predicament. After paying the proprietor of the motel, we dragged our luggage to the room, Image opened the door with great trepidation. The room was a starfish-themed nightmare! Everything was in shades of blue, green, and turquoise, with real and depicted starfish on nearly every Image

66.   F.  NO CHANGE

G.  it is

H.  that is

 J.  there is

67.   A.  NO CHANGE

B.  so

C.  we

D.  DELETE the underlined portion

68.   F.  NO CHANGE

G.  surface, the place looked like it hadn’t been redecorated since 1975!

H.  surface (looking like it hadn’t been redecorated since 1975).

 J.  surface; the place looked like it hadn’t been redecorated since 1975!

[1] With grim Image we shuffled across the somewhat gritty floor to further check out the place. [2] The couch was threadbare and lumpy and not exactly inviting. [3] The television was equipped Image that reminded me of the television that my grandpa Image in his basement workshop. [4] On the down side, however, the small kitchen table was so rickety that I was afraid to actually use it. [5] On the plus side, the room did have a kitchenette, so we could save money by cooking some meals inside. Image

69.   A.  NO CHANGE

B.  determined

C.  determination

D.  determine

70.   F.  NO CHANGE

G.  with a rusty, flimsy antenna

H.  with a rusty flimsy, antenna

 J.  with a rusty flimsy antenna

71.   A.  NO CHANGE

B.  will keep

C.  does keep

D.  keep

72.   For the sake of unity and coherence, Sentence 5 of this paragraph should be placed:

F.  where it is now.

G.  immediately before Sentence 2.

H.  immediately before Sentence 3.

 J.  immediately before Sentence 4.

Confronted with all of these problems, Sophie and I decided we had one option—to make the best of it and enjoy ourselves! We thought that it was about time to escape the pseudo-undersea atmosphere of the room and enjoy some real ocean views. Image

73.   The writer would like to conclude the final paragraph with a sentence that shows the shift in attitude she and her friend Sophie experienced. Which choice would best accomplish this?

A.  I begrudgingly accepted the fact that our motel room was terrible as we headed to the beach.

B.  As the old saying goes: “When life gives you lemons, make lemonade.”

C.  We headed to the beach moaning about our crazy motel room.

D.  I decided that my next spring break trip will definitely not be in Florida!

Questions 74 and 75 ask about the preceding passage as a whole.

74.   The writer is considering the addition of the following sentence to the essay:

I couldn’t help but be reminded of one of the most fascinating facts about starfish: that if you chop one up, a new starfish will grow from each remaining stump.

Given that this statement is true, should it be added to the essay, and if so, where?

F.  Yes, at the end of the second paragraph because the lady at the information booth mentioned the Starfish Inn. Adding the sentence would be an effective way for the writer to foreshadow the troubles she and her friend would soon have at the motel.

G.  Yes, at the end of the third paragraph, because the writer had just finished describing the starfish theme of the room.

H.  No, because it is evident that the writer is not interested in scientific facts.

 J.  No, because a scientific statement would be out of context in an essay describing the personal experiences of the writer and her friend.

75.   Suppose a travel agent hired the writer to write an article warning of the possible hazards of being unprepared for a vacation. Does this essay successfully fulfill the assignment?

A.  Yes, because the first paragraph clearly states that the writer and her friend traveled to Florida.

B.  Yes, because the essay gives an example of what can happen when you don’t make reservations before going on vacation.

C.  No, because the essay is primarily intended to be a humorous story about being forced to stay at a dilapidated motel.

D.  No, because the essay concerns college students and does not consider that others may also be unprepared for a vacation.

END OF THE ENGLISH TEST
STOP! IF YOU HAVE TIME LEFT OVER, CHECK YOUR WORK ON THIS SECTION ONLY.

Image

MATHEMATICS TEST

60 Minutes – 60 Questions

DIRECTIONS: Solve each of the problems in the time allowed, then fill in the corresponding bubble on your answer sheet. Do not spend too much time on any one problem; skip the more difficult problems and go back to them later. You may use a calculator on this test. For this test you should assume that figures are NOT necessarily drawn to scale, that all geometric figures lie in a plane, and that the word line is used to indicate a straight line.

1.   If x + 3 = n, then 2x + 6 = ?

A.  n + 3

B.  n + 6

C.  2n

D.  2n + 3

E.  2n + 6

2.   The expression a(b – 2c) is equivalent to:

F.  ab – 2a – 2c

G.  ab – 2ac

H.  ab – 2bc

 J.  ab – b – 2c

K.  ab – 2b – c

3.   Which 3 numbers should be placed in the blanks below so that the difference between consecutive numbers is the same?

__, 3, 10, __, 24 __

A.  –4, 17, 31

B.  0, 17, 30

C.  1, 13, 31

D.  2, 17, 25

E.  5, 15, 31

4.   Diane bought 1 DVD for $20.00 and 5 others that were on sale for $8.49 each. What was the average price per DVD that she paid for these 6 DVDs?

F.  Image

G.  Image

H.  Image

 J.  Image

K.  Image

5.   Roberto needs Image feet of lumber for a project. He has Image feet of lumber. How many more feet does he need?

A.  Image

B.  Image

C.  Image

D.  Image

E.  Image

6.   If x is a real number and 3x = 81, then 2x × 2 = ?

F.  4

G.  8

H.  16

 J.  32

K.  64

7.   A rectangular garden measures 60 feet by 25 feet. A fence completely encloses the garden. What is the length, in feet, of the fence?

A.  85

B.  170

C.  256

D.  625

E.  1,500

8.   If x = –6, then –x2 – 2x + 21 = ?

F.  –27

G.  –3

H.  21

 J.  45

K.  69

9.   The formula for the volume of a sphere is Image. If the radius, r, of a spherical ball is 2 inches, what is its volume, to the nearest cubic inch?

A.  8

B.  19

C.  25

D.  34

E.  96

10.   The expression 4c – 2d is equivalent to which of the following?

F.  4(c – 2d)

G.  2cd

H.  2(c – d)

 J.  4(c – d)

K.  2(2c – d)

11.   For each day on the job, you receive $20.00 plus a fixed amount for each lawn that you mow. Currently you are earning $95.00 per day for mowing 5 lawns. Today you will mow an additional 2 lawns. What will be your new daily earnings?

A.  $50.00

B.  $75.00

C.  $100.00

D.  $125.00

E.  $150.00

12.   Which of the following is a simplified form of 4x + 2x + y – x?

F.  3x + y

G.  5x + y

H.  2(x + 2)(x + y)

 J.  6x – y

K.  x(6 + y)

13.   When graphed in the standard (x,y) coordinate plane, which of the following equations does NOT represent a line?

A.  x = 3

B.  2y = 7

C.  –y = 2x + 1

D.  Image

E.  x2 = y – 7

14.   In the figure below, point C is the center of the circle. If a = 40°, what is the value of b?

Image

F.  80°

G.  70°

H.  60°

 J.  50°

K.  40°

15.   Which of the following solution sets has both x = 5 and x = 6 as solutions?

A.  (x – 6)(x + 5) = 0

B.  (x + 6)(x + 5) = 0

C.  (x + 6)(x – 5) = 0

D.  (x – 5)(x – 6) = 0

E.  x – 6 = x – 5

16.   If x = Image, then Image

F.  –4

G.  0

H.  1

 J.  2

K.  3

17.   As shown below, the diagonals of rectangle MNOP intersect at the point (5,–1) in the standard (x,y) coordinate plane. Point M is at (–1,–4). Which of the following are the coordinates for point O?

Image

A.  (–6,2)

B.  (–1,4)

C.  (9,3)

D.  (10,–3)

E.  (11,2)

18.   Tony is participating in a charity event and must collect pledges for every mile that he runs in the next 30 days. His friend pledges 9 cents per mile for the first 25 miles that he runs, and 7 cents per mile for each additional mile. Tony’s goal is to run 63 miles in the next 30 days. Assuming he meets but does not exceed his goal, what is the total amount Tony should collect from his friend?

F.  $2.25

G.  $4.91

H.  $6.66

 J.  $8.33

K.  $10.08

19.   If the inequality |x| > |y| is true, then which of the following must be true?

A.  x > 0

B.  x < y

C.  x = y

D.  x ≠ y

E.  x > y

20.   For which nonnegative value of x is the expression Image undefined?

F.  0

G.  5

H.  10

 J.  100

K.  400

21.   What is the slope-intercept form of – 3x + y + 8 = 0?

A.  y = –3x – 8

B.  y = – 3x + 8

C.  y = 3x – 8

D.  y = 3x + 8

E.  y = Image x + 8

22.   The two squares below have the same dimensions. The vertex of one square is at the center of the other square. What is the area of the shaded region, in square centimeters?

Image

F.  9

G.  12

H.  27

 J.  36

K.  72

23.   The lengths of the sides of a triangle are 3, 4, and 5 inches. What is the length, in inches, of the shortest side of a similar triangle that has a perimeter of 36 inches?

A.  6

B.  9

C.  12

D.  15

E.  18

24.   If 4(a + b) (a – b) = 40 and a – b = 20, then a + b = ?

F.  30

G.  20

H.  10

 J.  2

K.  Image

25.   The total daily profit, p, in dollars, from producing and selling x units, is given by the function p(x) = 17x – (l0x + c), where c is a constant. If 300 units were produced and sold last week for a profit of $1,900, then c = ?

A.  200

B.  100

C.  0

D.  –100

E.  –200

26.   If, for all x, (x7a – 2)3 = x57, then a = ?

F.  2

G.  3

H.  Image

 J.  Image

K.  57

27.   If 3 times a number n is added to 9, the result is negative. Which of the following gives the possible value(s) for n?

A.  –3 only

B.  0 only

C.  6 only

D.  all n < –3

E.  all n > –3

28.   One endpoint of a line segment in the (x,y) coordinate plane has coordinates (–5,3). The midpoint of the segment has coordinates (9,–1). What are the coordinates of the other endpoint of the segment?

F.  (–45,–3)

G.  (–14,4)

H.  (2,1)

 J.  (23,–5)

K.  (4,2)

29.   In the standard (x,y) coordinate plane, what is the radius of the circle (x – 3)2 + (y – 4)2 = 25?

A.  3

B.  4

C.  5

D.  16

E.  25

30.   In the right triangle pictured below, r, s, and t are the lengths of its sides. What is the value of tan α?

Image

F.  Image

G.  Image

H.  Image

 J.  Image

K.  Image

31.   For all Image

A.  Image

B.  3 – 4x

C.  Image

D.  Image

E.  Image

32.   In the figure below, lines m and n are parallel, lines o and p are parallel, and the measure of angle α is 40°. What is the measure of angle β?

Image

F.  40°

G.  50°

H.  110°

 J.  140°

K.  180°

33.   Which of the following degree measures is equivalent to 4.25π radians?

A.  270°

B.  360°

C.  594°

D.  765°

E.  945°

34.   Among the points graphed on the number line below, which is the closest to 1Image

Image

F.  A

G.  B

H.  C

 J.  D

K.  E

35.   The sides of a triangle measure 3Image meters, 3 meters, and 3 meters. What are the measures of the angles of the triangle, in degrees?

A.  30°–60°–90°

B.  90°–30°–30°

C.  40°–50°–90°

D.  90°–45°–45°

E.  45°–60°–90°

36.   What is the median of the data given below?

9, 13, 27, 22, 20, 31, 13

F.  13

G.  19

H.  20

 J.  21

K.  22

37.   If p is a positive integer that divides both 45 and 60, but divides neither 9 nor 10, what should you get when you add the digits in p?

A.  3

B.  2

C.  5

D.  6

E.  9

38.   What is the slope of any line perpendicular to the x-axis in the (x,y) coordinate plane?

F.  –1

G.  0

H.  1

 J.  Undefined

K.  Cannot be determined from the given information

39.   In the (x,y) coordinate plane, line m is perpendicular to the y-axis and passes through the point (5,–3). Which of the following is an equation for line m?

A.  x = 0

B.  x = 5

C.  y = –3

D.  y = x + 2

E.  y = x + 8

40.   If tan Image then sin β = ?

F.  Image

G.  Image

H.  Image

 J.  Image

K.  Image

41.   Jenny can walk 4 miles in (m + 3) minutes. At that pace, how many miles can she walk in 15 minutes?

A.  Image

B.  Image

C.  60(m + 3)

D.  Image

E.  Image

42.   Which of the following calculations will yield an even integer for any integer n?

F.  4n2

G.  3n2 + 1

H.  5n2 – 1

 J.  3n

K.  n2 – 2n

43.   In triangle CAB, the measure of ∠A is 45° and the measure of ∠B is 45°. If Image is 12 units long, what is the perimeter, in units, of triangle CAB?

A.  36

B.  36Image

C.  72

D.  24 + 12Image

E.  24 + 12Image

Use the following information to answer questions 44 and 45.

The table below shows the number of households in the town of Potterville, situated in Eaton County, with a highspeed Internet connection for each year from 1999 through 2006.

Image

44.   Which of the following years had the greatest increase in the number of households with a high-speed Internet connection over the previous year?

F.  2000

G.  2002

H.  2003

 J.  2005

K.  2006

45.   Census data shows that there were approximately 652 households in Eaton County with a high-speed Internet connection in 2000. According to this information, the number of Potterville households with a high-speed Internet connection was approximately what percent of the total number of households in Eaton County with a high-speed Internet connection in 2000?

A.  15%

B.  27%

C.  35%

D.  50%

E.  73%

46.   For what value of a would the following system of equations have an infinite number of solutions?

12x – 19y = 20
36x – 57y = 30a

F.  2

G.  3

H.  10

 J.  15

K.  50

47.   If logx 169 = 2, then x = ?

A.  2

B.  13

C.  84.5

D.  169

E.  338

48.   Let a Ξ b = (a + b)3 for all integers a and b. Which of the following is the value of 2 Ξ 4?

F.  8

G.  24

H.  64

 J.  216

K.  512

49.   What is the area of quadrilateral WXYZ if it has vertices with (x,y) coordinates W(2,4), X(5,4),Y(4,1), and Z(l,l)?

A.  Image

B.  6

C.  9

D.  8Image

E.  18

50.   In the standard (x,y) coordinate plane, if the x-coordinate of each point on a line is 3 more than twice the corresponding y-coordinate, the slope of the line is:

F.  –Image

G.  Image

H.  2

 J.  3

K.  6

51.   In the (x,y) coordinate plane, what is the radius of the circle having the points (–4,4) and (0,–2) as endpoints of a diameter?

A.  Image

B.  2Image

C.  Image

D.  2Image

E.  2Image

52.   If X, Y, and Z are real numbers, and XYZ = 1, then which of the following conditions must be true?

F.  Image

G.  X, Y, and Z > 0

H.  Either X = 1, Y = 1, or Z = 1

 J.  Either X = 0, Y = 0, or Z = 0

K.  Either X < 1, Y < 1, or Z < 1

53.   In the standard (x,y) coordinate plane, what is the y-intercept of the line 5x + y = 9?

A.  –9

B.  –5

C.  Image

D.  9

E.  45

54.   The average of a set of 7 integers is 24. When an 8th number is included in the set, the average of the set increases to 31. What is the 8th number?

F.  31

G.  55

H.  80

 J.  168

K.  217

55.   In the figure shown below, c = ?

Image

A.  75°

B.  70°

C.  65°

D.  60°

E.  55°

56.   The ratio of l to m is 3 to 4, and the ratio of p to m is 1 to 2. What is the ratio of l to p?

F.  6 to 1

G.  3 to 8

H.  3 to 2

 J.  3 to 1

K.  1 to 1

57.   Jordan has been hired to build a circular wading pool in his neighbor’s backyard. The rectangular backyard measures 40 feet wide by 70 feet long. Jordan’s neighbors want the pool to be as large as possible, with the edge of the pool at least 4 feet from the edge of the backyard all around. How long should the radius of the pool be, in feet?

A.  16

B.  32

C.  36

D.  40

E.  62

58.   Three distinct lines contained within a plane separate the plane into distinct regions. How many possible distinct regions of the plane may be separated by any 3 such lines?

F.  4, 7, 8

G.  4, 6, 7

H.  3, 6, 7

 J.  3, 5, 8

K.  3, 4, 6

59.   If the sum of the consecutive integers from –22 to n, inclusive, is 72, then n = ?

A.  94

B.  74

C.  50

D.  25

E.  23

60.   In a set of 13 different numbers, which of the following CANNOT affect the value of the median?

F.  Increasing the largest number only.

G.  Decreasing the largest number only.

H.  Increasing the smallest number only.

 J.  Increasing each number by 10.

K.  Doubling each number.

END OF THE MATHEMATICS TEST
STOP! IF YOU HAVE TIME LEFT OVER, CHECK YOUR WORK ON THIS SECTION ONLY.

Image

READING TEST

35 Minutes – 40 Questions

DIRECTIONS: This test includes four passages, each followed by ten questions. Read the passages and choose the best answer to each question. After you have selected your answer, fill in the corresponding bubble on your answer sheet. You should refer to the passages as often as necessary when answering the questions.

PASSAGE I

PROSE FICTION: Born in Paradise

Image

1.   When Granny says, “I’m not sure it will,” (line 38) she is expressing her concern that:

A.  schools in Barbados are dangerous.

B.  Martin is hyperactive and will likely behave poorly in school.

C.  school will not provide the academic challenge that Martin requires.

D.  life never gets any easier.

2.   It can be reasonably inferred from their conversation that Granny believes Sheila is:

F.  not as well-educated as Martin is.

G.  too overworked to recognize Martin’s gift.

H.  an incompetent parent.

 J.  overly solicitous with her sons.

3.   The idea that Martin’s mother is unaware of his abilities is best exemplified by which of the following quotations from the passage?

A.  “He even turns the pages. It’s very cute!”

B.  “Do you have any idea how much it costs to live there?”

C.  “Mom, think what you’re saying!”

D.  “I really appreciate you taking care of him like you have.”

4.   As it is used in line 36, the word it most nearly refers to the:

F.  library excursions.

G.  reading of books.

H.  taking care of Martin.

 J.  start of the school year.

5.   It can be inferred from the passage that Granny is:

A.  Martin’s paternal grandmother.

B.  Sheila’s mother-in-law.

C.  Martin’s maternal grandmother.

D.  an unknown wealthy benefactor.

6.   The passage makes it clear that Martin and his mother:

F.  plan to move to America.

G.  will remain distant.

H.  will be alienated from Martin’s brothers.

 J.  may never see Granny again.

7.   You may reasonably infer from the details in the passage that Sheila is:

A.  self-confident.

B.  in her early 20s.

C.  negligent in her care of her older sons.

D.  willing to do whatever it takes to help Martin succeed.

8.   You may reasonably infer from the passage that Martin’s brothers:

F.  attended a school for juvenile delinquents.

G.  mistreated Martin because they were jealous of his intelligence.

H.  were not well-liked in school.

 J.  cared more for themselves than for Martin.

9.   Within the passage, the main function of the second paragraph is to:

A.  describe the special bond between a grandmother and grandson.

B.  explain the deplorable living conditions of the main characters.

C.  provide evidence of a potential conflict in the story.

D.  introduce a comparison between an old woman and a young child.

10.   The title, “Born in Paradise,” combined with details presented in the passage implies that:

F.  everyone loves a tropical island.

G.  Caribbean islands tend to have subpar educational systems.

H.  children in single-parent homes need someone like Granny.

 J.  paradise is a relative term.

PASSAGE II

SOCIAL SCIENCE: The following passages discuss different battles of the Civil War.

Passage A

Passage B

Questions 11 – 13 ask about Passage A.

11.   As it is used in line 3, the word “engagement” most nearly means:

A.  match.

B.  commitment.

C.  arrangement.

D.  confrontation.

12.   In line 17 the phrase “so with great fanfare” suggests that General Lee was:

F.  broken and despondent.

G.  fatigued from recent battle.

H.  eager to promote his cause.

 J.  satisfied with his army’s progress.

13.   Which statement about the Army of Northern Virginia, if true, would most directly support the view described in lines 23–26?

A.  General Lee was a powerful but often barbaric man.

B.  Soldiers were difficult to control in times of great stress.

C.  Soldiers took great pains to keep their uniforms and equipment tidy.

D.  A rigid chain of command maintained strict order throughout the ranks.

Questions 14 – 17 ask about Passage B.

14.   As it is used in line 67, “grand” most nearly means:

F.  haughty.

G.  massive.

H.  wonderful.

 J.  extravagant.

15.   According to Passage B, Pickett’s Charge failed because of which of the following?

  I.  Artillery fire

 II.  Infantry charges

III.  Exhausted soldiers

A.  I only

B.  II only

C.  I and II

D.  I, II, and III

16.   It can reasonably be inferred from Passage B that the statement “the mile of men had narrowed to nearly half” (lines 99–100) suggests that the Confederate line:

F.  moved within firearms range.

G.  was suffering massive losses.

H.  was easy for Union soldiers to target.

 J.  made very slow progress across the field.

17.   In line 117, the quotation marks around the phrase “high water mark” serve to:

A.  indicate that this word is used allegorically.

B.  emphasize the limitations of military conquest.

C.  criticize the human preoccupation with expansion.

D.  emphasize the uniqueness of the author’s writing.

Questions 18 – 20 ask about both passages.

18.   Unlike the author of Passage A, the author of Passage B does which of the following?

F.  Details a specific battle.

G.  Explains a war strategy.

H.  Questions an officer’s decision.

 J.  Offers an alternative interpretation.

19.   Both Passage A and Passage B indicate that the Civil War:

A.  was amicably resolved.

B.  denied certain freedoms to Americans.

C.  resulted in many casualties on both sides.

D.  decided the borders of all fifty states in the Union.

20.   Which of the following most accurately describes a way in which the two passages are related to each other?

F.  F. Passage A introduces a topic that is further detailed in Passage B.

G.  Passage B suggests an alternate framework within which to interpret Passage A.

H.  The history suggested in Passage A is proven to be false by the facts in Passage B.

 J.  The evidence presented in Passage B serves to weaken the assertions made in Passage A.

PASSAGE III

HUMANITIES: The Passion of Perugino

21.   Which of the following descriptions most accurately and completely represents this passage?

A.  A reminiscent and passionate recollection of the narrator’s introduction to Perugino’s art

B.  An independent critical analysis of Monet, Renoir, and Manet in relation to Perugino

C.  An impartial evaluation of the paintings of Perugino

D.  A thorough biographical outline of Perugino’s life

22.   All of the following were unmistakably identified as painters in this passage EXCEPT:

F.  Leonardo da Vinci.

G.  Michelangelo.

H.  Botticelli.

 J.  Donatello.

23.   Which of the following quotations best expresses the main point of the passage?

A.  “Since that day in the museum, I have gained more knowledge and expertise about the Italian Renaissance movement, and I recognize that Pietro Perugino’s work is not beyond critique.”

B.  “I have seen the works of several painters from the Italian Renaissance that are considered far greater than anything created by Perugino.”

C.  “In this piece, Pietro Perugino showed how far art had come since the medieval times.”

D.  “In my mind, Perugino’s passion for art gives his pieces their distinction and this passion more than makes up for any deficiencies that the critics might find.”

24.   As it is used in the passage (line 26), the word ingenuity most nearly means:

F.  resourcefulness.

G.  inventiveness.

H.  quality.

 J.  versatility.

25.   It can be inferred from the passage that the narrator most highly values which of the following in an artist?

A.  Fluidity and volatility

B.  Unique appearance of subjects

C.  Devotion and passion for art

D.  Classical training from the masters

26.   It can be most reasonably concluded from the writer’s quote, “In that moment, my admiration for artists like Renoir and Manet of the French Impressionist Movement, was eclipsed by the austere exquisiteness of these fifteenth-century paintings,” (lines 14–18) that:

F.  few of the painters of the French Impressionist Movement were as impressive as the artists of the Italian Renaissance.

G.  the masters of the Italian Renaissance are more universally accepted than Renoir and Manet.

H.  the narrator believes that the technical skill and creativity of Perugino surpasses that of Renoir and Manet.

 J.  the narrator’s admiration of Perugino is so great, he or she believes that Perugino’s work outshines that of more well-known painters.

27.   According to the passage, what are characteristics of Perugino’s work?

I.  austerity

II.  showing the
passion of the artist

III.  ability to display depth

IV.  abstraction

A.  I, II, III only

B.  I, II only

C.  I, IV only

D.  I, II, IV only

28.   Which of the following best describes the narrator’s instant reaction upon seeing Perugino’s paintings for the first time?

F.  Disbelief in the quality of the work

G.  Unsettled by some of the features of the paintings

H.  Envious of Perugino’s genius and artistic ability

 J.  Intent on comparing Perugino’s work to French Impressionist artists

29.   All of the following are a criticism of Perugino’s paintings mentioned in the passage EXCEPT:

A.  Perugino’s paintings show a lack of imagination.

B.  Perugino’s technique in creating depth was not as advanced as Leonardo da Vinci’s.

C.  the subjects or people of Perugino’s paintings often look alike.

D.  Raphael could create more emotion in his paintings than Perugino.

30.   The narrator states his or her opinion about famous artists and their work throughout the passage. All of the following opinions are clearly stated in the passage EXCEPT:

F.  Manet’s work is reprinted on postcards and cheap posters because of its popularity.

G.  Leonardo da Vinci and Perugino could both display depth well.

H.  Botticelli’s work shows fluidity and ingenuity.

 J.  the painting Mona Lisa by Leonardo da Vinci shows subtle detail.

PASSAGE IV

NATURAL SCIENCE: The Prickly Porcupine

31.   The primary purpose of the passage is to:

A.  detail the various ways in which the quills of a porcupine are used by humans.

B.  give a brief overview of the porcupine, its habitat, and the misconceptions associated with it.

C.  prove false the “shooting-quills” legend associated with the porcupine.

D.  detail the safest way to remove porcupine quills from animals such as dogs and beavers.

32.   The author calls the porcupine a “threatening creature” (line 9) in the first paragraph because:

F.  it throws its quills at its enemies.

G.  no one can survive being attacked by a porcupine.

H.  it becomes alarmed and excited.

 J.  it has a frightening appearance.

33.   The passage indicates that, unlike some other wild animals, the porcupine:

A.  has a descriptive scientific name.

B.  does not have a defense mechanism.

C.  is not easily frightened.

D.  generally finds plenty of food.

34.   According to the author, the porcupine most likely moves slowly because:

F.  its quills add extra weight.

G.  it has no reason to move quickly.

H.  it has no predators.

 J.  it has short legs.

35.   Based on information in the passage, the author feels dogs are especially threatened by porcupines because:

A.  quills can cause great discomfort.

B.  dogs are likely to touch porcupines with their noses and mouths.

C.  porcupines routinely attack dogs.

D.  porcupines often wander into peoples’ backyards.

36.   The passage indicates that the Yukon government considers porcupines to be both:

F.  scarce and endangered.

G.  appreciated and useful.

H.  dangerous and unthreatening.

 J.  feared and disliked.

37.   Which of the following quotations best captures the second paragraph’s (lines 12–29) main focus?

A.  “Each of these quills … can imbed themselves into the flesh of a predator” (lines 14–17).

B.  “Rather than throwing their quills” (line 17).

C.  “Porcupines also swing their tails” (line 20).

D.  “Since these quills are hollow, they fill up with the host’s blood” (lines 22–23).

38.   The passage indicates that a porcupine turned over on its back would most likely be:

F.  safe to approach.

G.  killed by predators.

H.  unnoticed or ignored.

 J.  ready to attack.

39.   The passage indicates that, if imbedded, a porcupine’s quills:

A.  can cause death.

B.  can be very painful.

C.  should be left alone.

D.  will eventually fall out on their own.

40.   According to the passage, the scientific name for the porcupine means:

F.  “prickly animal.”

G.  “shooting quills.”

H.  “threatening creature.”

 J.  “irritable back.”

END OF THE READING TEST
STOP! IF YOU HAVE TIME LEFT OVER, CHECK YOUR WORK ON THIS SECTION ONLY.

Image

SCIENCE TEST

35 Minutes – 40 Questions

DIRECTIONS: There are seven passages in this test. Each passage is followed by several questions. You should refer to the passages as often as necessary in order to choose the best answer to each question. Once you have selected your answer, fill in the corresponding bubble on your answer sheet.

You may NOT use a calculator on this test.

 

PASSAGE I

A hurricane is a large, rotating storm centered around an area of very low pressure with strong winds blowing at an average speed in excess of 74 miles per hour. Hurricanes are dangerous natural hazards to people and the environment. However, they are also essential features of the Earth’s atmosphere. Hurricanes transfer heat and energy between the equator and the cooler regions toward the poles.

Two meteorologists present their views on hurricane formation.

Meteorologist 1

The most influential factors that turn a storm into a hurricane are a source of very warm, moist air coming from tropical oceans having surface temperatures greater than 26°C, and sufficient spin from the Earth’s rotation. The warm ocean heats the air above it, causing a current of very warm, moist air to rise quickly. This creates a center of low pressure at the surface. Trade winds rush in toward the area of low pressure, which force the inward-spiraling winds to whirl upward, releasing heat and moisture. The rotation of the Earth causes the rising column to twist. The rising air cools and quickly produces towering cumulus and cumulonimbus clouds. When the warm water evaporates from the tropical ocean, energy is stored in the water vapor. As the air rises, the majority of the stored energy is released as condensation, resulting in vertically growing cumulonimbus clouds and rain. The hurricane becomes a self-sustaining heat engine because the release of heat energy warms the air locally and causes a further decrease in pressure. Air rises faster to fill the low-pressure area and more warm, moist air is drawn off the sea. This gives the system additional energy. A hurricane causes major destruction when its path takes it over land, but this also leads to the destruction of the hurricane itself.

Meteorologist 2

Hurricanes start out as a group of storms that begin to rotate when they encounter converging winds. They are not necessarily formed where the surface temperature of the ocean is warm, nor does the Earth’s rotation have any bearing on whether storms turn into hurricanes. The converging winds spin the group of storms until they organize into a more powerful spiraling storm. The storm takes the form of a cylinder whirling around an “eye” of relatively still air. The spinning storm heats the surface of the ocean until the warm water turns into water vapor. The water vapor rises very quickly, rotating with the storms and helps to increase the wind speed. The cycle repeats itself and eventually the water vapor is released as condensation, resulting in a tremendous amount of rain. For the hurricane to die, it must reach land. The hurricane causes major destruction when it hits land, but it destroys itself at the same time. Over the past three decades, studies have shown that higher-than-average tropical ocean temperatures did not result in more hurricanes.

1.   Assuming that increased levels of atmospheric carbon dioxide (CO2) cause an increase in air temperatures, which of the following figures best represents the relationship between CO2 levels and the number of hurricanes, according to Meteorologist 1?

A. Image

B. Image

C. Image

D. Image

2.   Which of the following would Meteorologist 1 suggest leads to an increase in the number of hurricanes, assuming that increased levels of atmospheric CO2 cause an increase in air temperature?

F.  Decreased levels of atmospheric CO2

G.  Increased marine life in the oceans

H.  Decreased production and release of particles that increase water temperatures in tropical oceans

 J.  Reduced number of forests and trees that help remove CO2 from the atmosphere during photosynthesis

3.   It has been found that nontropical oceans and seas contain higher numbers of marine animals. This results in increased water temperatures at the surface due to the energy that is transferred from the movement of the marine life. However, hurricanes do not occur in these waters. This information would best support the view of:

A.  Meteorologist 1, because hurricanes are mostly self-sustaining.

B.  Meteorologist 1, because increased water temperatures cause hurricanes.

C.  Meteorologist 2, because increased water temperatures do not cause hurricanes.

D.  Meteorologist 2, because there are no converging winds in these waters.

4.   According to the hypothesis of Meteorologist 1, which of the following results is expected if global temperatures increase and water temperatures rise?

F.  There will be an increase in the number of hurricanes.

G.  There will be a decrease in the number of hurricanes.

H.  There will be no change in the number of hurricanes.

 J.  Hurricanes will be easier to predict.

5.   The views of both meteorologists are similar because they imply that:

A.  increased water temperatures alone help create hurricanes.

B.  converging winds are not a necessary component in hurricane formation and duration.

C.  hurricanes become stronger and more destructive with the presence of warm water and water vapor.

D.  there is a correlation between average water temperatures and the number of hurricanes.

6.   Meteorologist 2 states that higher:

F.  surface water temperatures cause hurricanes.

G.  surface water temperatures do not cause hurricanes.

H.  wind speeds do not cause hurricanes.

 J.  levels of CO2 cause hurricanes.

7.   The hypothesis of Meteorologist 2 could best be tested by:

A.  recording the surface temperature of nontropical oceans and seas over the next 10 years.

B.  recording the surface temperature of tropical oceans and seas over the next 10 years and comparing the data with the number of hurricanes recorded during the same time period.

C.  combining waters with cooler surface temperatures and high converging winds, and recording the data for at least 10 years.

D.  combining waters with warmer surface temperatures and high converging winds, and recording the data for at least 10 years.

PASSAGE II

Gas diffusion occurs when a gas moves from a syringe into a sealed vacuumed area where it can spread widely and thinly throughout the entire vacuumed area. A 50 ml gas sample in a syringe is forced into a sealed vacuumed area. The molecular mass in a.m.u. (atomic mass units), for 6 noble gases, as well as the time required for the gases to completely diffuse throughout the entire 10-cubic-foot (c3) vacuumed area, are recorded in Table 1. The densities (mass/volume), boiling point (Kelvin, or K), and melting point (Kelvin, or K) are also given.

Image

Note: Figures are all rounded to the nearest number.

Figure 1 shows a graph of the diffusion time versus the molecular mass of the noble gases.

Image

Figure 1

8.   Given the information in Table 1, the difference in diffusion times is greatest between which of the following pairs of noble gases?

F.  He and Kr

G.  Ne and Ar

H.  Kr and Rn

 J.  Ne and Xe

9.   Based on the passage, what is the relationship between molecular mass and diffusion time?

A.  Molecular mass increases as diffusion time increases.

B.  Molecular mass decreases as diffusion time increases.

C.  Molecular mass stays constant as diffusion time decreases.

D.  Molecular mass stays constant as diffusion time increases.

10.   Six vacuumed areas of identical shape and size are placed side by side. Six syringes, each containing 50 milliliters (ml) of one of the noble gases is forced into each vacuumed area. According to the data in Table 1, if these areas are kept under the same conditions, which gas should completely diffuse first?

F.  He

G.  Ne

H.  Ar

 J.  Kr

11.   A 50-ml sample of Ar gas is allowed to diffuse into a 10-cubic-foot vacuumed area. Given the information in Table 1, what percentage of the vacuumed area will NOT have Ar gas molecules after 14 seconds?

A.  12.25%

B.  25%

C.  50%

D.  75%

12.   According to Table 1, if a vacuumed area of unknown volume is filled with Ne gas, and it takes 54 seconds for this Ne gas to completely diffuse, then the volume of the vacuumed area will be closest to:

F.  2.5 cubic feet.

G.  5 cubic feet.

H.  20 cubic feet.

 J.  30 cubic feet.

PASSAGE III

A study was conducted to determine whether two processes (Process A and Process B) provided reliable data on the content of forest soil samples of varying acidity (acid concentration). The concentrations of several compounds commonly tested for in forest soil samples were measured. The results are presented in Table 1. The results for the processes were compared to estimates obtained using Standard Methods, which provide extremely accurate estimates.

Note: Higher pH levels indicate lower acidity.

13.   From the results of the study, one would conclude that at a pH level of 2, Process A is most accurate in measuring the concentration of which of the following compounds, relative to the Standard Method?

A.  Dissolved O2

B.  Dissolved CO2

C.  Calcium carbonate

D.  Dissolved CaCl2

14.   The data from which of the measurement procedures supports the conclusion that the concentration of calcium carbonate increases as the level of acidity decreases?

F.  Standard Method only

G.  Process A only

H.  Standard Method, Process A, and Process B

 J.  The data does not support the conclusion.

15.   Is the conclusion that Process A is more accurate than Process B for estimation of NH3 concentration supported by the results in the table?

A.  Yes, because the estimates using Process A are consistently lower than are the estimates using Process B.

B.  Yes, because the estimates using Process A are more similar to the estimates using the Standard Method than are the estimates using Process B.

C.  No, because the estimates using Process B are more similar to the estimates using the Standard Method than are the estimates using Process A.

D.  No, because the estimates using Process B are consistently higher than the estimates using Process A.

16.   Which of the following graphs best represents the relationship between pH level and the concentration of dissolved CO2 as estimated by Process A?

F.  Image

G.  Image

H.  Image

 J.  Image

17.   A forest soil sample of unknown acidity was tested using Process A. The concentrations, in milligrams per liter (mg/l), of selected compounds in this sample were: dissolved O2 = 5.5, dissolved CaCl2 = 72.1, and NH3 = 0.49. According to the data in the table, one would predict that the most likely pH level was:

A.  6

B.  5

C.  4

D.  2

PASSAGE IV

Pesticides are often used to kill and repel fleas and ticks on dogs. Two experiments were designed to measure the effectiveness of different pesticides on fleas and ticks. The pesticides work by making contact with the fleas and ticks during the application and by coating the dogs’ skin.

Experiment 1

A biologist tested two types of flea and tick pesticides on flea- and tick-infested dogs. Ten dogs were washed with a shampoo containing 8 oz. of Pesticide A, and 10 other dogs were washed with a shampoo containing 8 oz. of Pesticide B. All dogs weighed between 15 and 20 pounds (lbs) and were noted to have at least 10 visible ticks each. The shampoo volumes were identical. The dogs were then inspected for live fleas and ticks. The total number of remaining fleas and ticks were counted, averaged, and recorded 24 hours after treatment and 48 hours after treatment. The results are shown in Table 1.

Image

Experiment 2

A biologist tested three types of flea and tick shampoos: one containing only Pesticide A, one containing only Pesticide B, and a combination of the shampoos (Pesticide A + B), which contained 50% of Pesticide A and 50% of Pesticide B, on dogs with long and short coats. The experiment was conducted in the same manner as Experiment 1, except fleas and ticks were only counted and averaged after 24 hours. The results are shown in Table 2.

Information on average coat length of the dogs in both experiments is given in Table 3.

Image

18.   The results of Experiments 1 and 2 indicate that which type of pesticide was most effective in removing fleas?

F.  Pesticide A

G.  Pesticide B

H.  Pesticide A + B

 J.  Neither pesticide removed fleas.

19.   Which scenario would most reduce the number of ticks on a dog?

A.  Applying Pesticide B to a dog with a long coat

B.  Applying Pesticide A + B to a dog with a short coat

C.  Applying Pesticide B to a dog with a short coat

D.  Applying Pesticide A to a dog with a short coat

20.   Based on the results of Experiment 2, shorter coat length leads to:

F.  reduced effectiveness of all pesticides.

G.  increased effectiveness of Pesticide A only.

H.  increased effectiveness of all pesticides.

 J.  reduced effectiveness of Pesticide B only.

21.   Which of the following best explains why Pesticide A + B did not drastically reduce the number of both fleas and ticks?

A.  The two pesticides interfered with each other’s effectiveness.

B.  The outcome depended on whether the dog had a long or short coat.

C.  The pesticides did not remain in sufficient contact with the fleas and ticks.

D.  The combined volume of Pesticide A + B was less than that of the other pesticides.

22.   Assuming a coat length of 2 inches, what is the average number of ticks per dog 24 hours after application of Pesticide A?

F.  4

G.  7

H.  10

 J.  12

PASSAGE V

Radioactive decay is a natural process by which an atom of a radioactive isotope spontaneously decays into another element. The unstable nucleus disintegrates by emitting alpha or beta particles or gamma rays. This process changes the composition of the nucleus and continues to take place until a stable nucleus is reached. Half-life is the amount of time it takes for half of the atoms in a sample to decay.

Figure 1 shows the decay from Fluorine 22 to Neon 22.

Figure 1

Figure 2 shows the decay from Oxygen 22 to Fluorine 22 to Neon 22.

Figure 2

Table 1 shows the decay products and associated energy in MeV (million electron volts) and velocity, measured as a fraction of the speed of light.

Image

23.   According to Figure 1, what is the approximate half-life of Fluorine 22?

A.  16.9 seconds

B.  8.4 seconds

C.  4.2 seconds

D.  29.6 seconds

24.   Based on the passage, radioactive decay:

F.  is stable.

G.  does not occur in nature.

H.  is a natural process.

 J.  only occurs in half of the atoms.

25.   Based on Table 1, what is the relationship between decay energy and decay particle velocity?

A.  Lower decay energy leads to lower particle velocity.

B.  Lower particle velocity leads to higher decay energy.

C.  Decay energy does not impact particle velocity.

D.  Higher decay energy leads to lower particle velocity.

26.   When Cerium 53 decays into Lanthanum 127, the decay energy is 6.100 MeV. According to the data in Table 1, the decay particle velocity is most likely:

F.  greater than the particle velocity of Oxygen 22.

G.  approximately equal to the particle velocity of Flourine 22.

H.  greater than the particle velocity of Nitrogen 22.

 J.  approximately equal to the particle velocity of Oxygen 22.

27.   Based on Figure 2, at which time do Oxygen 22 and Neon 22 have the same percent of atoms remaining?

A.  2.2 seconds

B.  4.5 seconds

C.  6.7 seconds

D.  15.7 seconds

28.   What statement best explains the meaning of the shape of the Fluorine 22 curve in Figure 1 and the Oxygen 22 curve in Figure 2?

F.  Decay happens at a steady rate regardless of the number of atoms.

G.  Decay starts off slowly and then speeds up.

H.  Decay occurs very quickly at first and slows as the number of atoms is reduced.

 J.  The rate of decay is erratic.

PASSAGE VI

A study was conducted on the effects of pesticide exposure on domestic chicken breeds. Some zoologists believe that exposure to pesticides can lead to lower birth rates and increased susceptibility to illness. Table 1 shows the average number of eggs laid, average number of eggs that hatch, and resistance to illness before being exposed to a pesticide for several different domestic chicken breeds.

Image

Figure 1 shows how pesticide exposure affects the chicken’s susceptibility to illness.

Image

Figure 1

Figure 2 shows the average number of eggs laid and subsequent number of eggs that hatched after exposure to pesticide.

Image

Figure 2

29.   Based on the results of the study, if Chicken breed F has a low resistance to illness, approximately how many eggs would you expect to hatch from an average of 20 eggs laid, before pesticide exposure?

A.  5

B.  12

C.  17

D.  20

30.   Based on the information in Figure 1, which breed had the fewest illness incidences at the highest pesticide exposure [200 parts per million (ppm)]?

F.  Breed E

G.  Breed C

H.  Breed B

 J.  Breed A

31.   Based on the data in Figures 1 and 2, which breed is likely to have a higher number of eggs hatch per number of eggs laid, and have a low incidence of illness when exposed to pesticide?

A.  Breed B

B.  Breed A

C.  Breed E

D.  Breed C

32.   According to Table 1, what is the relationship between resistance to illness and average number of eggs laid?

F.  The average number of eggs laid remains constant.

G.  There is no direct relationship.

H.  As resistance increases, the average number of eggs laid increases.

 J.  As resistance increases, the average number of eggs laid decreases.

33.   According to Figure 1, which breed is the least affected by pesticide exposure?

A.  Breed A

B.  Breed B

C.  Breed D

D.  Breed E

34.   Based on the passage, which of the following statements is true?

F.  The average number of eggs hatched for all chicken breeds is not affected by pesticide exposure.

G.  Chicken breed C has the lowest resistance to illness.

H.  Chicken breed A had the lowest number of eggs hatch before exposure to pesticide.

 J.  The average number of eggs laid by all breeds is not affected by pesticide exposure.

35.   According to Table 1, which chicken breed laid the most eggs before pesticide exposure?

A.  E

B.  C

C.  B

D.  A

PASSAGE VII

Some students performed three studies to measure the average speed of a remote-controlled car on different surfaces. Each study was conducted on a fair day with no wind. A 100-foot-long flat surface was marked, and the car’s travel time was measured from start to finish with a stopwatch. The car was not modified in any way and the car’s batteries were fully charged before each trial.

Study 1

The students placed the car on a smooth asphalt road. One student started the car as the other student started the stopwatch. The student stopped the stopwatch as the car crossed the 100-foot mark. The students calculated the results of three separate trials and averaged the results (see Table 1).

Image

Study 2

The students repeated the procedure used in Study 1, except they placed the car on a rough gravel road. The results are shown in Table 2.

Image

Study 3

The students repeated the procedure used in Study 1, except they placed the car on a powdery, dry dirt road. The results are shown in Table 3.

Image

36.   The highest average speeds resulted from using which surface?

F.  Dirt road

G.  Gravel road

H.  Asphalt road

 J.  The speeds remained constant.

37.   According to Table 1, the average speed for all three trials is:

A.  greater than the speed measured in Trial 2.

B.  less than the speed measured in Trial 3.

C.  greater than the speed measured in Trial 1.

D.  equal to the speed measured in Trial 2.

38.   According to Tables 12, and 3:

F.  the average speed of a car on a gravel road is approximately Image of the average speed of a car on an asphalt road.

G.  the average speed of a car on a dirt road is approximately Image of the average speed of a car on a gravel road.

H.  the average speed of a car on an asphalt road is approximately twice the average speed of a car on a dirt road.

 J.  the average speed of a car on a gravel road is approximately twice the average speed of a car on a dirt road.

39.   Based on the passage, the lower average speeds were probably a result of:

A.  human error.

B.  greater friction.

C.  wind resistance.

D.  cloud cover.

40.   During which of the following was the travel time of the car the slowest?

F.  Study 1, Trial 2

G.  Study 2, Trial 2

H.  Study 2, Trial 3

 J.  Study 3, Trial 2

END OF THE SCIENCE TEST
STOP! IF YOU HAVE TIME LEFT OVER, CHECK YOUR WORK ON THIS SECTION ONLY.

WRITING TEST

DIRECTIONS: This test is designed to assess your writing skills. You have forty (40) minutes to plan and write an essay based on the stimulus provided. Be sure to take a position on the issue and support your position using logical reasoning and relevant examples. Organize your ideas in a focused and logical way, and use the English language to clearly and effectively express your position.

When you have finished writing, refer to the Scoring Rubrics discussed in Chapter 7 to estimate your score.

Note: On the actual ACT you will receive approximately 2.5 pages of scratch paper on which to develop your essay, and approximately 4 pages of notebook paper on which to write your essay. We recommend that you limit yourself to this number of pages when you write your practice essays.

SEAT BELT LAWS

Most states currently have laws on the books requiring some or all of the occupants of motor vehicles to fasten their seat belts while the vehicle is in motion. Penalties for operation of a motor vehicle while passengers are unrestrained range from simple fines to potential loss of driving privileges. In many states, there are exceptions for school buses and antique vehicles which were never equipped with seat belts. In some states, a vehicle may not be stopped by law enforcement merely for a seat belt violation, while in other states, officers may stop and ticket a driver for a seat belt violation alone.

Perspective One

Seat belt laws save lives. The statistics show that occupants who are properly restrained are much more likely to survive a serious accident than those who are not. Traffic fatalities nationwide have decreased significantly over the past few decades as seat belt laws have been passed by state after state.

Perspective Two

Seat belt laws are an infringement on our basic freedoms. Government does not have the right to force adult citizens to act responsibly simply for their own good. Each individual person is responsible for his or her own safety and society should not be permitted to penalize a person for taking a risk with his or her own body.

Perspective Three

Society must bear the medical expenses due to increased injury to individuals who do not use seat belts. Car accidents are still common, even with modern anti-lock brakes and other similar technologies. In many cases, automobile insurance will not cover treatment for injuries attributable to failure to use safety equipment, leaving all of us to foot the bill.

Essay Task

Write a unified, coherent essay in which you evaluate multiple perspectives on the implications of seat belt laws. In your essay, be sure to:

•   analyze and evaluate the perspectives given

•   state and develop your own perspective on the issue

•   explain the relationship between your perspective and those given

Your perspective may be in full agreement with any of the others, in partial agreement, or wholly different. Whatever the case, support your ideas with logical reasoning and detailed, persuasive examples.

Image   ANSWER KEY

English Test

1.   D

2.   F

3.   B

4.   G

5.   B

6.   H

7.   C

8.   H

9.   A

10.   J

11.   D

12.   J

13.   C

14.   H

15.   B

16.   J

17.   A

18.   F

19.   A

20.   F

21.   C

22.   J

23.   C

24.   J

25.   B

26.   F

27.   A

28.   H

29.   C

30.   F

31.   A

32.   G

33.   C

34.   F

35.   D

36.   F

37.   B

38.   J

39.   C

40.   H

41.   D

42.   F

43.   A

44.   J

45.   D

46.   G

47.   C

48.   F

49.   D

50.   F

51.   B

52.   H

53.   D

54.   F

55.   D

56.   J

57.   C

58.   G

59.   A

60.   H

61.   C

62.   G

63.   A

64.   G

65.   B

66.   H

67.   A

68.   J

69.   C

70.   G

71.   A

72.   J

73.   B

74.   J

75.   C

Mathematics Test

1.   C

2.   G

3.   A

4.   G

5.   C

6.   J

7.   B

8.   G

9.   D

10.   K

11.   D

12.   G

13.   E

14.   G

15.   D

16.   K

17.   E

18.   G

19.   D

20.   G

21.   C

22.   H

23.   B

24.   K

25.   A

26.   G

27.   D

28.   J

29.   C

30.   J

31.   E

32.   J

33.   D

34.   K

35.   D

36.   H

37.   D

38.   J

39.   C

40.   F

41.   D

42.   F

43.   D

44.   G

45.   B

46.   F

47.   B

48.   J

49.   C

50.   G

51.   C

52.   F

53.   D

54.   H

55.   C

56.   H

57.   A

58.   G

59.   D

60.   F

Reading Test

1.   C

2.   G

3.   A

4.   H

5.   C

6.   F

7.   D

8.   J

9.   C

10.   J

11.   A

12.   F

13.   B

14.   J

15.   C

16.   F

17.   D

18.   H

19.   B

20.   G

21.   A

22.   J

23.   D

24.   G

25.   C

26.   J

27.   A

28.   G

29.   B

30.   F

31.   B

32.   J

33.   C

34.   G

35.   B

36.   G

37.   A

38.   G

39.   B

40.   J

Science Test

1.   A

2.   J

3.   C

4.   F

5.   C

6.   G

7.   B

8.   J

9.   A

10.   F

11.   C

12.   J

13.   A

14.   H

15.   C

16.   F

17.   D

18.   G

19.   D

20.   H

21.   A

22.   G

23.   C

24.   H

25.   A

26.   J

27.   B

28.   H

29.   B

30.   H

31.   A

32.   G

33.   D

34.   J

35.   A

36.   H

37.   C

38.   F

39.   B

40.   H

Image   SCORING GUIDE

Your final reported score is your COMPOSITE SCORE. Your COMPOSITE SCORE is the average of all of your SCALED SCORES.

Your SCALED SCORES for the four multiple-choice sections are derived from the Scoring Worksheet on the next page. Use your RAW SCORE, or the number of questions that you answered correctly for each section, to determine your SCALED SCORE. If you got a RAW SCORE of 60 on the English test, for example, you correctly answered 60 out of 75 questions.

Step 1   Determine your RAW SCORE for each of the four multiple-choice sections:

Image

The following Raw Score Table shows the total possible points for each section.

Multiple-Choice Scoring Worksheet

Step 2   Determine your SCALED SCORE for each of the four multiple-choice sections using the following Scoring Worksheet. Each SCALED SCORE should be rounded to the nearest number according to normal rules. For example, 31.2 ≈ 31 and 31.5 ≈ 32. If you answered 61 questions correctly on the English section, for example, your approximate SCALED SCORE would be 29.

If you take the optional Writing Test, you should refer to Chapter 7 for guidelines on scoring your Writing Test Essay.

Step 3   Determine your COMPOSITE SCORE by finding the sum of all your SCALED SCORES for each of the four sections: English, Mathematics, Reading, and Science, and divide by 4 to find the average. Round your COMPOSITE SCORE according to normal rules. For example, 31.2 ≈ 31 and 31.5 ≈ 32.

Image   ANSWERS AND EXPLANATIONS

English Test Explanations

PASSAGE I

1.   The best answer is D.  The underlined portion must be a verb with tense, or else the sentence would be a fragment. Therefore, eliminate answer choices B and C. Answer choice D, which has past tense, is best because the sentence clearly states that Noh began in Medieval Japan, a past time period.

2.   The best answer is F.  This question requires you to select the correct punctuation for the underlined portion. Answer choice G has unnecessary commas. Both answer choice H and answer choice J improperly use semicolons. The comma after Zeami is necessary because the two independent clauses are conjoined with and.

3.   The best answer is B.  The sentence references a time period in the past (fourteenth and fifteenth centuries). To describe an action (in this case, staying unchanged) that began in the past and is ongoing in the present, the present perfect tense is appropriate. Only answer choice B uses this verb tense.

4.   The best answer is G.  The words only, just, and solely all have similar meaning. To avoid redundancy, you should use only one of them in the sentence. Eliminate answer choices FH, and  J.

5.   The best answer is B.  The interrupting phrase for example should always be set off by commas when it appears within a sentence. The only answer choice that places a comma before and after for example is answer choice B.

6.   The best answer is H.  The noun being replaced by the pronoun in this sentence is instrumentalists, which is plural. Therefore, you must use the plural pronoun they. Eliminate answer choices F and J. The noun scores is the direct object of the verb play, so no preposition (for) is necessary. Eliminate answer choice G.

7.   The best answer is C.  Paragraph 3 discusses the stylized masks worn by the actors to reflect certain characters. The sentence that best introduces this topic is answer choice C.

8.   The best answer is H.  The actors, not the characters that they portray, wear the masks, so you can eliminate answer choice F. The word with suggests that the audience is wearing the masks; eliminate answer choice G. In English idiom, someone recognizes something or someone by some feature or characteristic.

9.   The best answer is A.  This question requires you to select an answer choice that discusses a unique talent. Answer choice A explains that tilting their heads is a specific skill that must be learned, so it is the best selection. The other answer choices either are too general or they include information about the masks, not the actors.

10.   The best answer is J.  A semicolon must be immediately followed by an independent clause or a phrase that starts with a conjunctive adverb such as therefore. Eliminate answer choice F because the semicolon is not followed by an independent clause. The items in a list must be separated by commas if there are three or more items in the list. The only remaining choice with correct comma usage is answer choice  J.

11.   The best answer is D.  In context, the adjective abstract means nonspecific or somewhat difficult to define and understand. While an abstract is a summary of a text, speech, and so on, this definition is not appropriate based on the context.

12.   The best answer is J.  The most important clue indicating the correct tense is the adverb since, which denotes a duration of time beginning in the past. Eliminate answer choice G because it refers to the future. In this case, the time period extends to the present. It did not end in the past, so the past perfect tense is not appropriate here. Eliminate answer choice F, which is a present perfect passive-voice construction. In passive constructions, as in H and J, the past participle is used. The past participle of write is written, not wrote, which is the simple past form. Eliminate answer choice H.

13.   The best answer is C.  First, decide whether you should use its or it’s. In this sentence, the noun Noh theater is being replaced by the pronoun it. The performers belong to the theater, so you should use the possessive form of it, which is its. Eliminate answer choices A and D. There is a contrast suggested in the second half of the sentence, so the correct conjunction is but, making answer choice C correct.

14.   The best answer is H.  The fact mentioned in this sentence exists now, so eliminate answer choice G, which has future tense, and answer choice J, which has past tense. Second, the subject must agree with the verb. In this sentence, the subject is the long noun clause the fact that it has remained essentially in its original form for over 600 years. Although this noun clause ends with a plural noun, the central, controlling noun that determines its grammatical number is fact, which is singular. Therefore, a singular verb is needed, speaks, answer choice H.

15.   The best answer is B.  The sentence contains information on Zeami, one of the original playwrights. Zeami is not discussed in either Paragraph 2 or Paragraph 3, so eliminate answer choices A and C. Since you are left with Paragraph 5, decide whether the sentence should be placed after Sentence 1 or Sentence 3. As Sentence 1 mentions Zeami, it would make sense to place the new sentence after Sentence 1.

PASSAGE II

16.   The best answer is J.  The items in a series must be separated by commas. While answer choice G contains the correct number of commas, it omits the conjunction and, which is essential to the sentence.

17.   The best answer is A.  The preceding sentence mentions computer fonts. It is appropriate to provide a transition into the rest of the passage that is concerned with calligraphy as an art form.

18.   The best answer is F.  The verb derive (here as a past participle in a passive-voice construction) can take as a complement to a prepositional phrase beginning with from, in which case it means originate (from…). This subject of the sentence, calligraphy, has its origins in the two Greek words kalli and graphia.

19.   The best answer is A.  The sentence as it is written is clear and concise and in the active voice. The other answer choices are awkward.

20.   The best answer is F.  This question requires you to best express the idea that legible handwriting was important and useful in many places. Answer choices H and J create incomplete sentences and should be eliminated. Between answer choice F and G, answer choice G can be eliminated because it says in two words what answer choice F says in one word.

21.   The best answer is C.  The sentence is describing a general property (the age) of Chinese calligraphy. Therefore, the simple present tense is appropriate. Eliminate answer choices B and D, which have past tense. The subject Chinese calligraphy is third-person, singular; therefore, the verb must be third-person, singular: dates, answer choice C.

22.   The best answer is J.  This is a passive-voice sentence in which Chinese scholars is the agent (who or what does the action of the verb). With active voice, Chinese scholars would be the subject. Recall that making sentences passive usually results in moving the subject to the end of the sentence and after the preposition by. This is the function of by required in this underlined portion. Eliminate answer choices F and H. Next, recognize that for use byis idiomatic, whereas for the use by is not. Therefore, answer choice J is best. If you cannot recognize that idiom, select answer choice J because it is more concise.

23.   The best answer is C.  According to the passage, the scribes started using the index around 200 B.C., which is clearly in the past. Therefore, you should use the simple past form of the verb develop. Eliminate answer choices A (present perfect) and B (a gerund). Eliminate answer choice D because it begins a relative clause, leaving the clause without a verb.

24.   The best answer is J.  This question requires you to express the idea clearly and concisely. It is implied in the paragraph that the scribes replicate the characters when the scribes use the index. Therefore, it is not necessary to include any phrase about replicating the characters. If you omit the underlined portion, neither the sentence nor the paragraph lose any meaning.

25.   The best answer is B.  This question requires you to choose the best conjunctive adverb. A conjunctive adverb can be used to join two independent but related ideas, and is often used at the beginning of a sentence, if that sentence is related to the one directly preceding it. The conjunctive adverbs however, yet, and otherwise suggest a contrast that doesn’t exist in this paragraph. It makes sense that soon after the scribes developed their own, individual styles, the scribes would emerge as artists.

26.   The best answer is F.  The subject of the sentence, the Japanese, is followed directly by the verb adapted. Do not separate the subject from the verb with a comma. Eliminate answer choices G and H. Answer choice J does not contain a verb with tense, so eliminate it, too.

27.   The best answer is A.  It is idiomatic in this context to say appreciation for.

28.   The best answer is H.  This question requires you to express the idea clearly and concisely. First, determine whether it is the Church or the calligraphy that is being influenced. Based on the context of the passage, it makes sense that the calligraphy is being influenced. Eliminate answer choices G and J, which suggest that calligraphy influenced the Church. It is better to use the active voice, as in answer choice H, which clearly indicates that the Church influenced calligraphy.

29.   The best answer is C.  To maintain parallelism within this sentence, the adjective phrase closely spaced must be in the comparative form (more closely spaced) to match the comparative form narrower. Therefore, answer choice C is correct.

30.   The best answer is F.  The singular subject equipment requires a singular verb. Remember that subject and verb must match in tense.

PASSAGE III

31.   The best answer is A.  The myth that is the subject of this sentence exists in the present, so the simple present tense is appropriate. Eliminate answer choices C and D. The myth is singular, so use the singular verb paints, answer choice A.

32.   The best answer is G.  To avoid redundancy, use only the verb participating in this sentence. The remaining answer choices are awkward and redundant.

33.   The best answer is C.  The primary focus of the first paragraph is the mountain man, not the beaver pelts that he harvested. Therefore, the sentence would not be a relevant addition to the paragraph. Answer choice D is not correct because the statement is off-topic.

34.   The best answer is F.  There is no punctuation required in this phrase that includes two adjectives describing the same noun.

35.   The best answer is D.  The clearest and most concise way of expressing the idea is with buyers. Answer choice A is wordy, so eliminate it. Using buyers with product is redundant, so eliminate answer choices B and C.

36.   The best answer is F.  It makes sense that the mountain men would need goods other than the food they captured. (They may have needed clothes, ammunition, cooking utensils, etc.) The other answer choices contain information that is outside the scope of the passage.

37.   The best answer is B.  It is important to maintain parallelism within the sentence. So, the subject and verb must have the same form. Since the subject, mountain man, is singular, the verb must also be singular. Eliminate answer choices A and C. The other verb forms in the sentence, was and relied, are past tense, so eliminate answer choice D, which includes the present-tense verb appears.

38.   The best answer is J.  This question requires you to recognize that consumer is used as the first noun of a compound with the noun demand. Any form of the verb demand would create an ungrammatical sentence. Therefore, answer choices F, G, and H can be eliminated.

39.   The best answer is C.  The logical opposite of freelancers is being an employee of a firm. Only answer choice C expresses that some trappers were employed by a particular fur company.

40.   The best answer is H.  With commas, provided that all grammar rules are followed, fewer is better. In this sentence, the passive voice verb construction were called has the noun phrase Men hired directly by a fur company as its subject. The noun Men is modified by the past participle phrase hired directly by a fur company, so no comma should separate them. Eliminate answer choices F, G, and  J.

41.   The best answer is D.  The sentence already says that the furs were company property, so it is not necessary to include any more information about to whom the furs did or did not belong. Answer choices A, B, and C are all redundant and should be eliminated.

42.   The best answer is F.  A semicolon should be followed by an independent clause that provides more information about the first part of the sentence. The sentence is correct as written. You should not use a comma to separate two main clauses. This is known as a comma splice. Eliminate answer choice H. It is necessary to include some form of punctuation, so eliminate answer choice J. By removing the word he in answer choice G, an incomplete sentence is created.

43.   The best answer is A.  The sentence structure suggests a cause-and-effect relationship. The phrase because of provides the proper connection between the effect and the cause. The phrase in regards with is not idiomatic in standard written English. Eliminate answer choice B. Answer choice C is missing an initial with, so eliminate it. Irregardless is not a word and should never be used, so eliminate answer choice D.

44.   The best answer is J.  The paragraph is in the past tense, therefore it is appropriate to use the past tense verb succumbed. The sentence as it is written uses the present tense verb; answer choices G and J use the “-ing” form of the verb, which is incorrect in this context.

45.   The best answer is D.  This question requires you to determine the main idea of the essay. The essay introduces the concept of the myth of the mountain man, and then goes on to describe the reality of living as a mountain man, which was quite different. Answer choice D best supports the ideas presented in the essay.

PASSAGE IV

46.   The best answer is G.  It is idiomatic to return home from some place. Answer choice F says from a severe case of tonsillitis, so it can be eliminated. Answer choices H and J can be eliminated because the relative pronoun which and the verb was make them wordy. In addition, answer choices H and J attribute, respectively, the tonsillitis to playing football and playing football to the tonsillitis.

47.   The best answer is C.  It is important to maintain parallelism within the paragraph. The verbs returned, expressed, and convinced are all past tense. Therefore, a past-tense verb should be used in the underlined portion. Eliminate answer choices A and D. Eliminate answer choice B because it is awkward. The simplest way to express the idea conveyed in the sentence is to use recommended, answer choice C.

48.   The best answer is F.  This question requires you to express the idea clearly, concisely, and in the correct word order. First, determine who is the logical subject of the verb called. Fans and players had their own nickname for the team. Answer choice F is best because it has active voice. Answer choice H has passive voice, so it can be eliminated. Answer choice J is awkward; it, too, can be eliminated.

49.   The best answer is D.  The modifiers present-day, contemporary, and up-to-date all have the same meaning, so none of them can be the correct answer. Since the sentence already includes the word modern, it would be redundant to include any of the answer choices. Therefore, omit the underlined portion, answer choice D.

50.   The best answer is F.  This question requires you to express the idea clearly and concisely. Answer choice H suggests that they were already wearing uniforms and had to change into different uniforms at home. This is not supported by information in the paragraph, so eliminate answer choice H. Answer choice G separates the verb changed from its prepositional phrase complement into their uniforms, so it is somewhat awkward and can be eliminated. Answer choice J is wordy and can be eliminated.

51.   The best answer is B.  In this sentence, so is a coordinating conjunction joining two independent clauses. Independent clauses joined with coordinating conjunctions must be separated by a comma placed immediately before the conjunction. Eliminate answer choices A and D. When two nouns are joined with a coordinating conjunction (here, gates and bleachers), no comma should be used. Therefore, eliminate answer choice C.

52.   The best answer is H.  Idiomatically, the noun way can take a verb in the infinitive form (to + bare form) as a complement. In this case, that is the clearest and most concise way to express the intended idea.

53.   The best answer is D.  The phrase quite literally is an interrupting phrase; therefore, it should be set apart from the sentence using commas.

54.   The best answer is F.  The first sentence as it is written adequately introduces the main idea of the paragraph (the Packers’ humble beginnings) and does not need to be replaced. While the conditions under which the Packers played football during the first year were difficult, the paragraph does not support the idea that the conditions were brutal, so eliminate answer choice G. The other answer choices are not supported by the context of the paragraph.

55.   The best answer is D.  By definition, a game is played; therefore, answer choices A, B, and C are redundant and can be eliminated.

56.   The best answer is J.  This question asks you to find a way to link information already given in the passage with the information that is to follow. Since the paragraph introduces the historic game played at Hagemeister Park and indicates that it was the first game that the Packers played as professionals, answer choice J makes the most sense. The other answer choices refer specifically to individuals or contain irrelevant information.

57.   The best answer is C.  The adjective named is appropriate to precede a building’s proper name, here Lambeau Field. Answer choice A can be eliminated for wordiness. Answer choices B and D are awkward structurally and logically and can be eliminated.

58.   The best answer is G.  According to the passage, Lambeau Field was named after a person named Lambeau. Founder in answer choice G correctly and succinctly refers to this man.

59.   The best answer is A.  The verb come takes a prepositional object beginning with from and is modified by the phrase a long way. Therefore, no commas are needed between any of these elements. Eliminate answer choice B. Answer choice C can be eliminated because a semicolon joins dependent clauses. The verb pass takes a prepositional indirect object (to whom or to what place the direct object passes). No comma must separate verb from object, so eliminate answer choice D.

60.   The best answer is H.  This question requires you to determine the main idea of the essay. The essay focuses primarily on the beginnings of the Green Bay Packers and some of the team’s success and doesn’t really have anything to do with any economic influence the team may have had on the city of Green Bay.

PASSAGE V

61.   The best answer is C.  The sentence following the sentence containing the underlined portion explains how the author and her friend ended up at the Starfish Inn. The conjunction because of implies that the reason they ended up at the Starfish Inn was their own irresponsibility. Answer choice C is the clearest choice.

62.   The best answer is G.  The coordinate conjunction and suggests that, in addition to it being their freshman year of college, the friends wanted to get away for spring break. Yet and but suggest a contrast that doesn’t exist, so eliminate answer choices F and H. The word where suggests a specific location; freshman year of college is not a location, so eliminate answer choice  J.

63.   The best answer is A.  To maintain parallelism in this paragraph, verbs must have the same form. Arrived, tried, delivered, and so on are past forms. Answer choice D has future tense, so eliminate it. Answer choice C is wordy and awkward, so eliminate it. Answer choice B uses the passive voice, and it is awkward in this sentence.

64.   The best answer is G.  The underlined portion should mean “obtain.” This eliminates answer choice F. Answer choices H and J have similar meanings, but they are too literal to be used with lodging. Only answer choice G has the correct meaning and is appropriate to use with lodging or other services.

65.   The best answer is B.  The adjective limited modifying budget implies that the budget is small. Therefore, the sentence as written is redundant. Eliminate answer choice A. The second sentence in each of answer choices C and D is irrelevant to the topic of the passage, so they can be eliminated.

66.   The best answer is H.  First, eliminate answer choice F because theres does not exist. Second, there refers to a location, so eliminate answer choice J. Answer choice G is awkward because the pronoun it lacks a clear antecedent. Answer choice H is best because that can refer to what the reader just read.

67.   The best answer is A.  The word where indicates a location. The girls dragged their luggage to the room (the location) and then opened the door. Answer choices B and D create incomplete sentences and should be eliminated. Answer choice C creates a comma splice, so it can be eliminated.

68.   The best answer is J.  This question requires you to select the correct punctuation, while maintaining the meaning of the sentence. A semicolon should be followed by an independent clause, which is the case in answer choice J. Since The place looked like it hadn’t been redecorated since 1975! is an independent clause, you cannot use a comma. This creates what is called a comma splice, so eliminate answer choice G. No coordinating conjunction (so) should be used with a semicolon to join independent clauses. Eliminate answer choice F. Using parentheses as in answer choice H complicates the sentence and takes emphasis away from the writer’s exclamatory reaction to the décor of the hotel room.

69.   The best answer is C.  The adjective grim must modify a noun. Answer choices A, B, and D can be eliminated because they are all verb forms.

70.   The best answer is G.  Rusty and flimsy are coordinate adjectives, meaning they modify antenna in a similar way. Coordinate adjectives can be separated using and or a comma. Answer choices H and J have neither, so eliminate them. However, no comma should come between the modifiers and the noun, so eliminate answer choice F.

71.   The best answer is A.  The verb reminded suggests that the television was kept in the past. Eliminate answer choices B (future), C (present emphatic), and D (bare or present plural form).

72.   The best answer is J.  It makes the most sense to place Sentence 5 immediately before Sentence 4, because Sentence 5 introduces the kitchenette and Sentence 4 provides some additional information about the kitchenette. The sentence would be inappropriate placed anywhere else in the paragraph.

73.   The best answer is B.  The writer and her friend are originally very disappointed with the condition of the motel room. However, they decide to make the best of it and enjoy themselves. This suggests that they took a bad situation and turned it into a good one. The selection that best acknowledges this shift is answer choice B.

74.   The best answer is J.  This question requires you to identify the main idea of the essay. The essay is primarily about the difficulties that the friends encountered on their trip and how they ended up staying at a subpar motel. Even though the motel was called the Starfish Inn, any information included in the passage about the animal starfish would be irrelevant.

75.   The best answer is C.  Although the essay does provide an example of what could go wrong if you don’t make reservations before going on vacation, it does not fully discuss possible hazards of being unprepared for a vacation. The essay is a humorous account of being forced to stay at a dilapidated motel, answer choice C.

Mathematics Test Explanations

1.   The correct answer is C.  To answer this question, solve the first equation for x:

x + 3 = n

x = n – 3

Next, substitute n – 3 for x in the second equation:

2(n – 3) + 6

= 2n – 6 + 6

= 2n

2.   The correct answer is G.  This question tests your ability to recognize and apply the distributive property. According to the distributive property, for any numbers a, b, and c, c(a + b) = ca + cb. If you distribute the a value, you get ab – a2c, or ab – 2ac.

3.   The correct answer is A.  The first step in solving this problem is to determine what the difference is between the consecutive numbers. You are given 2 consecutive numbers, 3 and 10, which differ by 7. Think of the numbers as being on a number line. Since the first number must be 7 units from the number 3, and the numbers are in ascending order, the first number must be 7 units to the left of 3 on the number line. Count backwards 7 units from 3 and you will arrive at –4. Since only answer choice A includes –4 in the first blank, answer choice A must be correct.

4.   The correct answer is G.  To find the average price that Diane paid per DVD, you must divide the total dollar amount that Diane paid for the DVDs by the number of DVDs that Diane bought. The total dollar amount that Diane paid for the DVDs can be set up like this:

1 DVD for $20.00 + 5 DVDs for $8.49 each

$20.00 + 5($8.49)

You know from information in the problem that Diane purchased a total of 6 DVDs. Divide the total dollar amount that she paid, $20.00 + 5($8.49), by 6:

Image

5.   The correct answer is C.  One way to solve this problem is to convert feet into inches. There are 12 inches in 1 foot, so Roberto needs (18 × 12) + 3, or 219 inches of lumber. He currently has (10 × 12) + 6, or 126 inches of lumber. Therefore, he needs 219 – 126, or 93 inches of lumber. 93 inches of lumber. 93 ÷ 12 = 7.75, which is equivalent to Image feet.

6.   The correct answer is J.  The first step in solving this problem is to determine the value of x. You know that 32 = 9, and 92 = 81, so 34 must equal 81. Therefore, x = 4. Now, substitute 4 for x in the second equation and solve: 24= 16 and 16(2)=32.

In addition, because you know that 32 = 9, you know that x must be greater than 2, and you can eliminate answer choices F and G. This process of elimination will help you to narrow down the answer choices if you are not sure how to arrive at the correct answer.

7.   The correct answer is B.  According to the problem, the fence completely encloses the garden. This means that it goes all the way around the garden. Therefore, the length of the fence must be equal to the perimeter of the garden. One formula for calculating the perimeter of a rectangle is 2l + 2w. Plug the numbers from the problem into this formula:

2(60) + 2(25)

= 120 + 50

= 170

8.   The correct answer is G.  Simply plug –6 in for x wherever x appears in the equation and solve the equation. Don’t forget to keep track of the negative signs!

–(–62) – 2(–6) + 21

= –(36) – (–12) + 21

= –36 + 12 + 21

= –3

9.   The correct answer is D.  Substitute the value for the radius given in the problem, 2, into the equation and solve:

Image

You also need to know that π is approximately equal to 3.14. Multiply 10.67 by 3.14 and round: 10.67 × 3.14 = 33.5, which means that the volume, to the nearest cubic inch, is 34, answer choice D.

10.   The correct answer is K.  This problem tests your ability to recognize and apply the distributive property; however, you must work backward. According to the distributive property, for any numbers a, b, and c, c(a + b) = ca + cb. In this problem, since 2 is the common factor for both 4 and 2, you can “factor out” 2. Eliminate answer choices F and J, which have incorrectly factored out 4. Once you factor out 2, the expression will look like this: 2(2c – d).

11.   The correct answer is D.  The first step in solving this problem is to calculate the amount of money that you earn each day for mowing lawns:

   $95.00 (total amount earned per day)
– $20.00 (fixed amount earned per day)
= $75.00 (amount earned for lawns mowed).

Next, calculate the amount that you earn per lawn that you mow:

$75.00 (amount earned for lawns mowed) ÷ 5 (number of lawns mowed) = $15.00 (amount earned per lawn mowed).

Now determine the amount that you will earn today for mowing the extra lawns:

2 (additional number of lawns mowed) × $15.00 (amount earned per lawn mowed) = $30.00 (additional income for the day).

Finally, add this amount to your current daily earnings:

$95.00 + $30.00 = $125.00

12.   The correct answer is G.  In the expression 4x + 2x + y – x, 4x, 2x, and –x are like terms and can be added together:

4x + 2x + (– x) = 5 x

The term with x and the term with y cannot be added because they contain different variables, so the simplified form of 4x + 2x + y – x is 5x + y.

13.   The correct answer is E.  The slope-intercept form of the equation of a line is y = mx + b. If y equals 0, and the slope of the line is 1, then x = 3 could be the equation of a line, so eliminate answer choice A. Answer choice E is actually the equation for a parabola, which is NOT a line, so answer choice E is correct.

14.   The correct answer is G.  To solve this problem, first recall that the total measure of the interior angles of a triangle is 180°. Also, because the 2 sides of the triangle originating from the center of the circle are each equivalent to the radius of the circle, the sides are congruent. This means that the angles opposite those sides are also congruent. You can set up an equation like the one shown next to solve for b:

a + 2b = 180

40 + 2b = 180

2b = 140

b = 70

15.   The correct answer is D.  To solve this equation, set each element of the equations in the answer choices equal to 0 and solve for x. When you get the solutions 5 and 6, that will be the correct answer.

(x – 6) = 0; x = 6 and (x + 5) = 0; x = –5; eliminate answer choice A.

(x + 6) = 0; x = –6 and (x + 5) = 0; x = –5; eliminate answer choice B.

(x + 6) = 0; x = –6 and (x – 5) = 0; x = 5; eliminate answer choice C.

(x – 5) = 0; x = 5 and (x – 6) = 0; x = 6; answer choice D is correct.

16.   The correct answer is K.  To solve this problem, substitute 1/2 for x wherever it appears in the equation:

Image

Remember that 1 ÷ 1/2 is equivalent to 1 × 2.

= 2 + 2 – 1

= 3

17.   The correct answer is E.  The diagonals cross at the midpoint of line MO, which means that point O is as far away from the point (5,–1) as M is. Starting with the x-coordinates, 5 –(–1) = 6, the distance from the midpoint to Mon the x-axis. The y-coordinates are –1 – (–4) = 3, the distance from the midpoint to M on the y-axis. The coordinates of point O, then, is x = 5 + 6, or 11 and y = –1 + 3, or 2. Point O is located at (11,2).

18.   The correct answer is G.  The first step in solving this problem is to calculate the amount of money Tony’s friend will donate for the first 25 miles that Tony runs:

25 miles × $0.09 = $2.25

Next, calculate the amount of money Tony’s friend will donate for the remaining miles:

63 miles (Tony’s goal) – 25 miles = 38 miles

38 miles × $0.07 = $2.66

Now, add the 2 amounts together to get the total:

$2.25 + $2.66 = $4.91

19.   The correct answer is D.  The only instance in which the absolute value of x could possibly be greater than the absolute value of y is when x is not equal to y, answer choice D. If x = y, then |x| cannot be greater than |y|.

20.   The correct answer is G.  An expression is undefined when the denominator equals 0. Set the denominator equal to 0 and solve for x:

100 – 4x2 = 0

100 = 4x2

25 = x2

5 = x

21.   The correct answer is C.  The slope-intercept form is expressed as y = mx + b. The equation given is –3x + y + 8 = 0. Isolate y on the left side of the equation: y = 3x – 8.

22.   The correct answer is H.  The best approach to this problem is to extend the sides of the shaded square into the nonshaded square, as shown below:

Image

By doing this you will see that the shaded region is Image of one of the squares. Since the squares have the same dimensions, calculate the area of the shaded square: Area of a square = side2. Each side is equal to 6 centimeters, so the area is 62, or 36 square centimeters. Multiply the total area of the square, 36, by Image to get the area of the shaded region: Image which is 27, answer choice H. Once you determined that the shaded region was Image of the total area, 36, you could have eliminated answer choices F and G as being too small, and answer choices J and K as being too big, leaving you with answer choice H.

23.   The correct answer is B.  Similar triangles have the same shape and the same proportions. The perimeter of the first triangle is 3 + 4 + 5, or 12 inches. You are given that a similar triangle has a perimeter of 36, which is 3 times the perimeter of the first triangle. Therefore, each side in the second triangle must be 3 times the length of the corresponding side in the first triangle. Since the shortest side of the first triangle is 3 inches, the shortest side of the second triangle must be 3 x 3, or 9 inches.

24.   The correct answer is K.  This first step in solving this problem is to simplify the equation by dividing both sides by 4:

4(a + b)(a – b) = 40

(a + b)(a – b) = 10.

Next, substitute 20 for a – b and solve for a + b:

Image

25.   The correct answer is A.  To solve this problem, simply substitute the given values into the equation, as follows:

p(x) = 17x – (10x + c)

1,900 = 7x – c

1,900 = 7 × 300 – c

1,900 = 2,100 – c

c = 200

26.   The correct answer is G.  When exponents are raised to an exponential power, the rules state that you must multiply the exponents by the power to which they are raised. In this problem, x is raised to the (7a – 2) power. This exponent is then cubed, so you should multiply 7a – 2 by 3: 3(7a – 2) = 21a – 6. You now have the equation x21a – 6 = x57. Since the bases are equal (x), the exponents must also be equal, so 21a – 6 = 57. Solve for a:

21a – 6 = 57

21a = 63

a = 3

27.   The correct answer is D.  In order for the result to be negative, 3n must be less than 9. When you add any negative number larger than 9 to 9, the result will be negative. Therefore, n must be less than –3.

28.   The correct answer is J.  To solve this problem you should use the Midpoint Formula. The midpoint of a line, M, is equal to the average of the x-coordinates and the average of the y-coordinates. The formula looks like this:

Image

You are given 1 point on the line (–5,3) and the midpoint of the line (9, –1). Since the midpoint is (9, –1) the average of the x-coordinates is 9, and the average of the y-coordinates is – 1. Set up equations to solve for the other endpoint:

Image

The x-coordinate of the other endpoint is 23. Since only answer choice J includes an x-coordinate of 23, it must be the correct answer. If you solve for the y-coordinate in the same way that you solved for the x-coordinate, you will get –5.

29.   The correct answer is C.  A circle centered at (a,b) with a radius r, has the equation (x – a)2 + (y – b)2 = r2. Based on this definition, a circle with the equation (x – 3)2 + (y – 4)2 = 25 would have a radius of Image. If r2 = 25, then r = Image, or 5.

30.   The correct answer is J.  The tangent of any acute angle is calculated by dividing the length of the side opposite the acute angle by the length of the side adjacent to the acute angle Image In this problem, the length of the side opposite angle α is r, and the length of the side adjacent to angle α is s. Therefore, the tan of angle α is Image

31.   The correct answer is E.  When you subtract fractions you must first find the common denominator. Multiply the denominators to get 4x as the common denominator, then solve for x:

Image

32.   The correct answer is J.  The figure in the problem represents 2 parallel lines cut by 2 parallel transversals. The angles created as a result have special properties. Where each of the parallel lines is cut by a transversal, there are 2 pairs of vertical, or opposite angles. Each angle in the pair is congruent to, or equal to, the other angle in the pair. Therefore, where m cuts o and also where it cuts p, two 40° angles are formed, which means that angle α = 40°; in addition, two 140° angles are formed that are adjacent to the 40° angles, since a straight line has 180°. So, since the same angles are created where n cuts o and p, and angle β is opposite of the 140° angle that is adjacent to angle α, angle β must be equal to 140°.

33.   The correct answer is D.  Because there are 2π radians in the circumference of every circle and a circle consists of 360°, π radians = 180°. Divide 4.25π radians by π to get 4.25; multiply 4.25(180°) to get the degree measure of the angle, 765°.

34.   The correct answer is K.  In order to solve this problem you must recognize that 1Image is exactly halfway between 1.5 (which equals 1Image)and 2 on the number line. This means that the point closest to 1Image on the number line is point E.

35.   The correct answer is D.  By definition, the legs of a 45°–45°–90° have the same length, and the hypotenuse is Image times as long as either leg. Since you are given that the length of 2 legs is 3 meters, and the length of the third leg, the hypotenuse, is 3Image meters, this must be a 45° –45° –90° triangle, answer choice D. Also, since the measure of the angles in a triangle must equal 180°, you can eliminate answer choices B and E.

36.   The correct answer is H.  The median is the middle value in a list that is in either ascending or descending order. Your first step is to put the data in order, as follows:

9, 13, 13, 20, 22, 27, 31

Because the list includes an odd number of values, simply pick the middle value which is 20.

37.   The correct answer is D.  To solve this problem, first list all of the distinct factors of 45: 1, 3, 5, 9, 15, 45. All of these numbers divide evenly into 45. Next, list all of the distinct factors of 60: 1, 2, 3, 4, 5, 6, 10, 12, 15, 30, 60. All of these numbers divide evenly into 60. The only factors that both 45 and 60 have in common are 1, 3, 5, and 15. Since you are told that p is NOT a factor of either 9 or 10, you can eliminate 1, 3, and 5, which factor evenly into either 9 or 10. This leaves you with a value for p of 15. When you add the digits (1 + 5) you get 6.

38.   The correct answer is J.  The slope of a line is defined as the change in the y-values over the change in the x-values in the standard (x,y) coordinate plane. Slope can be calculated by using the following formula: ImageAny line perpendicular to the x-axis is a vertical line: The x values do not change (see diagram).

Image

The slope of a vertical line is undefined, answer choice J, because there is no change in x, which means that the denominator (x1 – x2) is 0.

39.   The correct answer is C.  To solve this problem, it is helpful to draw a picture like the one shown below:

Because you are given that the line is perpendicular to the y-axis, you know that the y-intercept must be –3.

40.   The correct answer is F.  The tangent of any acute angle is calculated by dividing the length of the side opposite the acute angle by the length of the side adjacent to the acute angle Image The sine of any acute angle is calculated by dividing the length of the side opposite the acute angle by the hypotenuse Image In this problem, the tangent of angle β is Image. This means that the length of the side opposite angle β is 3 units, and the length of the side adjacent to angle β is 4 units. Therefore, by definition, the sine must be 3 units (the length of the side opposite angle β) over some number greater than 4, since the hypotenuse is always the longest side. The only answer choice that will work is Image.

41.   The correct answer is D.  This problem requires you to set up a simple proportion and solve for a variable. According to information in the problem, Jenny can walk 4 miles in m + 3 minutes. This means that she can walk 4 miles per m + 3 minutes, or Image The question asks you to calculate the number of miles that she can walk in 15 minutes. In other words, Jenny can walk x miles per 15 minutes, or Image what is the value of x? Set up a proportion and solve for x:

Image

42.   The correct answer is F.  The best approach to this problem is to pick some numbers for n, substitute them into the answer choices, and eliminate the answer choices that do not always yield an even number:

F: If n = 1, then 4n2= 4(1)2 = 4, which is even. If n = 2, then 4n2= 4(2)2 = 16, another even number. Because you are multiplying n2 by 4, an even number, the result will always be even. Answer choice F is correct. Check the other answer choices:

G: If n = 2, then 3n2 + 1 = 3(2)2 + 1 = 12 + 1 = 13, which is odd. Eliminate answer choice G.

H: If n = 1, then 5n2 = 5(1)2 = 5, which is odd. Eliminate answer choice H.

J: If n = 1, then 3n = 3(1) = 3, which is odd. Eliminate answer choice J.

K: If n = 3, then n2 – 2n = (3)2 – 2(3) = 3, which is odd. Eliminate answer choice K.

Answer choice F is the only choice that will always give you an even number for any value of n.

43.   The correct answer is D.  The perimeter of a triangle is calculated by adding together the lengths of all 3 sides. Based on the measures of the angles given, you can draw triangle CAB as shown below:

Image

You are given that Image, one of the legs, is 12 units long. Because this is a 45°-45°-90° triangle, the length of the other leg, Image is also 12 units long. In a 45°-45°-90° triangle, the hypotenuse is Image times longer than either leg. Therefore, the length of the hypotenuse is 12Image. Add together the lengths of all 3 sides to find the perimeter:

12 + 12 + 12Image = 24 + 12 Image

44.   The correct answer is G.  The best approach to this question is to test each answer choice:

F: The increase from 1999 to 2000 was 176 – 152, or 24.

G: The increase from 2001 to 2002 was 422 – 231, or 191.

H: The increase from 2002 to 2003 was 516 – 422, or 94.

J: The increase from 2004 to 2005 was 780 – 647, or 133.

K: The increase from 2005 to 2006 was 825 – 780, or 45.

The increase from 2001 to 2002 was the greatest, so answer choice G is correct.

45.   The correct answer is B.  To solve this problem, first look at the table to see that there were 176 households in Potterville that a had high-speed Internet connection in 2000. Next, set up a ratio comparing the number of households to the percent of households, as follows:

Image

The number of Potterville households with a high-speed Internet connection was approximately 27% of the total number of households in Eaton County with a high-speed Internet connection in 2000.

46.   The correct answer is F.  Systems of equations will have an infinite number of solutions when the equations are equal to each other. The first step in solving this problem is to recognize that the second equation is exactly 3 times the value of the first equation: 36x = 3(12x), 57y = 3(19y), so 30a must equal 3(20). Solve for a:

30a = 3(20)

30a = 60

a = 2

47.   The correct answer is B.  Logarithms are used to indicate exponents of certain numbers called bases. This problem tells you that log to the base x of 2 equals 169. By definition, loga b = c if ac = b. So, the question is, when x is raised to the power of 2, you get 169; what is x? By definition, logx 169 = 2 when x2 = 169. The square root of 169 is 13.

48.   The correct answer is J.  The question states that the operation a Ξ b = (a + b)3 for all integers a and b. Therefore, if a = 2 and b = 4, then (2 + 4)3 = 63 = 216.

49.   The correct answer is C.  One way to solve this problem is to plot the points and draw a figure like the one shown below:

Image

After plotting the points, you see that the figure is a parallelogram, whose area is equal to (b)(h). The base is equal to 3, and the height is equal to 3. Therefore, the area is 3 x 3, or 9.

50.   The correct answer is G.  You can express the phrase the x-coordinate is 3 more than twice the corresponding y-coordinate as follows: x = 2y + 3. The slope-intercept form for the equation of a line is y = mx + b, where m is the slope. Put the equation in the slope-intercept form:

Image

51.   The correct answer is C.  The length of the diameter is equal to Image or Image or Image which can be simplified to Image The radius is then half the length of that diameter, or Image

52.   The correct answer is F.  If XYZ = 1, then Z cannot equal 0. If Z (or X or Y, for that matter) were 0, then XYZ would equal 0. Both sides of the equation can be divided by Z, which gives you Image, answer choice F. Answer choice G is incorrect because 2 of the values could be – 1. Answer choice H is incorrect because 2 of the values could be fractions and the third value could be a whole number, that, when multiplied by the fractions equals 1.

53.   The correct answer is D.  The slope-intercept form of a line is y = mx + b, where m is the slope and b is the y-intercept. Put the equation given in the problem in the slope-intercept form:

5x + y = 9

y = –5x + 9; the y-intercept is 9.

54.   The correct answer is H.  If the average of 7 integers is 24, then the total must be 7 · 24, or 168. If the average of 8 integers is 31, then the total must be 8 · 31, or 248. Since you are adding an 8th integer to the set, the value of the 8th integer will be the difference between 248 and 168: 248–168 = 80, answer choice H.

55.   The correct answer is C.  To solve this problem, first recall that the total measure of the interior angles of a triangle is 180°. It might be helpful to fill in values for the right angles, as shown below:

Image

Next, set up an equation to solve for b:

2b + 90 + 40 = 180

2b = 180 – 130

b = 25

Now, set up an equation to solve for c, substituting 25 for b:

b + c + 90 = 180

25 + c + 90 = 180

b = 180 – 115 = 65

56.   The correct answer is H.  You are given that l/m = Image and p/m = Image. The ratio of l/p is equivalent to Image x Image, or Image, which can be reduced to Image.

57.   The correct answer is A.  First, draw the picture of the wading pool according to the information given in the problem, where the distance from the edge of the pool to the edge of the long side of the rectangular region is 4 feet. The distance from the edge of the pool to the edge of the short side of the rectangular region can be anything greater than 4, but it is not necessary to know this distance to solve the problem:

Image

Now you can determine the diameter of the circular pool. The diameter is the maximum distance from 1 point on a circle to another (the dashed line). Since the short side of the rectangular region is 40 feet, and the distance from the edge of the circular pool to each edge of the long sides of the rectangular region is set at 4 feet, the diameter of the circle must be 40 feet – 2(4 feet), or 40 feet – 8 feet, or 32 feet. The question asks for the radius of the pool, which is Image of the diameter. 32 ÷ 2 = 16.

58.   The correct answer is G.  To solve this problem, start by drawing 3 parallel lines.

Image

This creates 4 distinct regions, so the minimum number of distinct regions must be 4. Eliminate answer choices H, J, and K, which give the minimum number of distinct regions as 3. Now, try drawing 3 lines in other configurations, and you will see that there will always be either 6 or 7 regions:

Image

Therefore, the correct answer is 4, 6, or 7 distinct regions, answer choice G.

59.   The correct answer is D.  To solve this problem, you can apply some logic: because each number between – 22 and 22 will cancel each other out, you can start with the next consecutive integer, 23. Now, simply begin adding consecutive integers until you reach 72: 23 + 24 + 25 = 72. Therefore, n must equal 25.

You can also solve this problem mathematically by using the following formula:

Image

By expanding this equation and simplifying it, you can reach the equation (2n + 1)2 = 2,601. Therefore, 2n + 1 = 51 and n = 25.

60.   The correct answer is F.  The median is the middle number in an ordered list of numbers. Therefore, the value of the median can be changed by increasing each number by 10 or by doubling each number, so eliminate answer choices J and K. Likewise, if you increase the smallest number or decrease the largest number, you could potentially change the order of the numbers in the list, thereby potentially changing the value of the median; eliminate answer choices G and H. However, if you increase the largest number, it will still remain in the last position in the list, so the value of the median will not change.

Reading Test Explanations

PASSAGE I

1.   The best answer is C.  Granny makes this statement in response to Sheila’s comment that, once Martin goes to school, “it will all be so much easier.” This suggests that Granny was not convinced that school would challenge Martin effectively. The other answer choices are not supported by the passage.

2.   The best answer is G.  During the conversation Granny says, “I know you’re tired from working long hours, but Martin isn’t reciting—he’s reading!” You can infer that Granny believes Sheila is too overworked to recognize Martin’s gift, answer choice G.

3.   The best answer is A.  When Martin’s mother says, “He even turns the pages. It’s very cute!” she indicates that she doesn’t actually think that he can read. This is the best example of the idea that she is unaware of his abilities. The other answer choices do not reflect Sheila’s ignorance of her son’s reading ability.

4.   The best answer is H.  The context indicates that Sheila and Granny are having a discussion about Granny’s care of Martin, and that “it” will get easier when Martin starts school.

5.   The best answer is C.  Throughout the passage, Granny is clearly referred to as Martin’s grandmother, so answer choice D should be eliminated. It is also made clear that Granny is Sheila’s mother. Therefore, since Sheila is Martin’s mother, Granny must be Martin’s maternal grandmother, answer choice C.

6.   The best answer is F.  At the end of the passage, Martin’s mother agrees that she will start finding out how to get to America with Martin. This best supports answer choice F.

7.   The best answer is D.  Sheila states in the passage, “I’ll start finding out what I need to do to get Martin and me to America.” This suggests that she understands the importance of going to America and will probably do whatever she can to help Martin succeed. The other answer choices are not supported by the details in the passage.

8.   The best answer is J.  The first paragraph states that Martin’s brothers are selfish boys, who “let Martin fend for himself” and “made him the target of their pranks.” This suggests that Martin’s brothers cared more for themselves than they did for Martin. The other answer choices are not supported by the passage.

9.   The best answer is C.  The second paragraph shows the trouble that Granny has taking care of Martin because she is old and lacks the energy to keep up with him. This is a conflict in the story because Granny must now decide upon another course of action. The rest of the story tells about Granny’s resolution of this conflict.

10.   The best answer is J.  The first paragraph states that “Martin spent most of his childhood in a tropical paradise” but that his “early life was difficult.” The word paradise most often refers to a delightful or beautiful place. In this case, however, despite the fact that the island of Barbados contains the natural beauty associated with a paradise, the living conditions were not so delightful. This best supports answer choice J.

PASSAGE II

11.   The best answer is D.  The word “engagement” can refer to an “encounter or a battle.” The context of the passage supports answer choice D.

12.   The best answer is H.  The word “fanfare” suggests a “display” of some sort. The passage indicates that Lee wanted to take advantage of an opportunity to advance into Northern territory.

13.   The best answer is D.  According to the passage, “Soldiers obeyed Lee’s order to refrain from violence and pillaging, so for several days the townspeople maintained tacit compliance and sold food, clothes, and shoes to Southern troops.” These details best support answer choice D.

14.   The best answer is G.  The author uses the word “grand” to describe the large loss of life in the Battle of Gettysburg. “Massive” means “large in scale, amount, or degree,” so it is the best choice.

15.   The best answer is D.  The passage mentions “devastating artillery bombardment,” “battle fatigued soldiers,” and “infantry assault” as reasons for the failure of Pickett’s Charge.

16.   The best answer is G.  By using the phrase “mile of men,” the author implies that there were many soldiers. When the author says that “the mile of men had narrowed nearly to half,” he is indicating that the number of soldiers had been significantly reduced during the battle.

17.   The best answer is A.  Allegory suggests symbolism. Because the author is not speaking of literal water, but instead is using the phrase “high water mark” to refer to the greatest number of Confederate soldiers, he is using the phrase allegorically.

18.   The best answer is F.  Passage A mentions several battles or engagements but does not describe any specific battle, whereas Passage B focuses primarily on a description of the Battle of Gettysburg.

19.   The best answer is C.  One of the themes throughout both passages is the large loss of life that resulted from the many battles fought during the Civil War.

20.   The best answer is F.  Passage A introduces the topic of Civil War battles and the great number of casualties that resulted. Passage B expands upon that notion with a description of one particular battle.

PASSAGE III

21.   The best answer is A.  The narrator states in the introduction that “the unworldly experience continues to haunt my memory as I recall the unflinching gazes of Pietro Perugino’s subjects staring blankly at me as I admired the power and beauty of the great Italian Renaissance master’s most famous works of art.” Although the narrator briefly speaks about the work of other painters in relation to Perugino, this is not the focus of the paragraph. The narrator is also not impartial about the work of Perugino. In fact, the narrator admits that Perugino is one of his favorite painters. The passage also tells very little about Perugino’s life outside of his contributions to art. Eliminate answer choices B, C, and D.

22.   The best answer is J.  The passage does not mention Donatello as a painter. However, all of the other answer choices are explicitly identified as painters in the passage.

23.   The best answer is D.  Throughout the passage the narrator discussed his or her strong feelings about Perugino’s art. The answer choice that best supports this main idea is answer choice D.

24.   The best answer is G.  Ingenuity is another word for creativity or inventiveness. The narrator is acknowledging that despite his or her admiration for Perugino’s work, it lacked the originality of Botticelli.

25.   The best answer is C.  The narrator states, “Although the brilliance of all of the Italian Renaissance masters is undeniable, the aweinspiring beauty of Michelangelo’s work or the subtle detail of da Vinci’s Mona Lisa cannot match the simple passion evident in Perugino’s paintings.” Even though Perugino’s work may not show some of the technical skill of other Renaissance painters, the narrator believes Perugino’s work outshines the more complicated pieces because of his devotion to and passion for his craft.

26.   The best answer is J.  The use of the word eclipsed suggests that the work of Renoir and Manet was overshadowed by Perugino’s work.

27.   The best answer is A.  All of the following quotes appear in the passage: “In that moment, my admiration for artists like Renoir and Manet of the French Impressionist Movement, was eclipsed by the austere exquisiteness of these fifteenth-century paintings,” “cannot match the simple passion evident in Perugino’s paintings,” and “Perugino also experimented with depth, and he rivaled Leonardo da Vinci in his ability to create a definite background and foreground.” These statements best support answer choice A. Abstraction is not mentioned in the passage.

28.   The best answer is G.  The narrator writes, “I remember feeling slightly disconcerted as I looked up at the unsmiling saints, the Virgin Mary, and even Jesus as I wandered through the hushed halls of the museum.” Disconcerted is a synonym for unsettled.

29.   The best answer is B.  The passage states that “Perugino also experimented with depth, and he rivaled Leonardo da Vinci in his ability to create a definite background and foreground.” This suggests that he was as talented as Leonardo da Vinci at creating depth, which is not a criticism.

30.   The best answer is F.  The passage states that “The work of da Vinci and Michelangelo is seen on postcards and reprinted on cheap posters everywhere because of its universal appeal.” The author does not mention Manet in the discussion about postcard and poster reprints.

PASSAGE IV

31.   The best answer is B.  The passage begins with the author presenting general information about the porcupine and its behavior; this is followed by a discussion of its habitat and concludes by talking about the ways in which porcupines are useful to humans. Answer choice B contains the broadest survey of the information presented within the passage.

32.   The best answer is J.  The first paragraph states that “the porcupine’s mere outward appearance provides more than adequate reason for it rarely to become alarmed or excited.” The passage goes on to describe the porcupine as a “threatening creature,” which suggests that its appearance is what makes it threatening. The other answer choices are not supported by the passage.

33.   The best answer is C.  According to the passage, the porcupine, “unlike most other animals in the wild,” has a threatening appearance that allows it to remain unexcited in the face of danger. The other answer choices are not supported by details in the passage.

34.   The best answer is G.  The author’s statement that “the porcupine’s mere outward appearance provides more than adequate reason for it rarely to become alarmed or excited” suggests that the porcupine moves slowly because it has no reason to move quickly, answer choice G.

35.   The best answer is B.  Information in the passage indicates that “a noseful or mouthful of porcupine quills” can cause “excruciating pain.” This best supports answer choice B.

36.   The best answer is G.  According to the passage, the Yukon Department of Environment considers the porcupine useful, and believes that it has been and can be “appreciated by many.” The passage goes on to give examples of the utility of the porcupine quill. This best supports answer choice G.

37.   The best answer is A.  The topic of the second paragraph is the danger of the porcupine’s quills—specifically that these quills can become imbedded and cause serious pain. This idea is best summed up in answer choice A.

38.   The best answer is G.  According to the passage, some animals “are able to break down the porcupine’s powerful defense system by carefully turning the porcupine over onto its back, exposing its soft and vulnerable underbelly. Bobcats, cougars, and coyotes are especially adept at this technique and pose a major threat to the porcupine.”

39.   The best answer is B.  According to the passage, if a dog gets a noseful or mouthful of quills, it should be “tended to right away,” so eliminate answer choice C. The passage goes on to say that extraction of the quills can “relieve the excruciating pain,” so it makes sense that imbedded porcupine quills can be very painful, answer choice B.

40.   The best answer is J.  The scientific name for the porcupine is Erethizon dorsatum, which is Latin for “irritable back.” The other answer choices are mentioned in the passage, but not in reference to the scientific name of the porcupine.

Science Test Explanations

PASSAGE I

1.   The correct answer is A.  Meteorologist 1 believes that the presence of very warm air is one of the things that most influences hurricane formation. Since higher levels of CO2 increase air temperatures, it is likely that Meteorologist 1 would suggest a direct relationship between CO2 levels and the number of hurricanes. The graph in answer choice A shows a direct relationship—as CO2 levels increase, so does the number of hurricanes.

2.   The correct answer is J.  Meteorologist 1 believes that higher air temperatures contribute to hurricane formations. It is given that increased levels of atmospheric CO2 cause an increase in air temperature. Therefore, reducing the number of forests and trees, which remove CO2 from the atmosphere, would lead to higher levels of CO2 in the atmosphere, higher air temperatures, and an increased number of hurricanes.

3.   The correct answer is C.  According to Meteorologist 2, higher water temperatures do not necessarily lead to hurricane formation. Therefore, any evidence suggesting that hurricanes do not occur in areas of the ocean with higher water temperatures would support Meterologist 2’s viewpoint. This best supports answer choice C.

4.   The correct answer is F.  Meteorologist 1 believes that higher air and water temperatures contribute to hurricane formation. Therefore, it is likely that Meteorologist 1 would predict an increase in the number of hurricanes if both air and water temperatures increased. The other answer choices are not supported by Meteorologist 1’s viewpoint.

5.   The correct answer is C.  Based on the passage, both meteorologists believe that hurricanes become stronger and more destructive with the presence of warm water and water vapor. While Meteorologist 2 does not believe that higher water temperatures cause hurricanes, the passage indicates that Meteorologist 2 does believe that higher water temperatures increase water vapor levels, which leads to an increase in wind speed.

6.   The correct answer is G.  Meteorologist 2 states that hurricanes “are not necessarily formed where the surface temperature of the ocean is warm,” which best supports answer choice G.

7.   The correct answer is B.  Meteorologist 2 suggests that higher water temperatures are not a factor in hurricane formation. A good way to test this theory would be to record surface temperatures of tropical oceans and seas over time, and compare that data with the number of hurricanes recorded during the same time period, answer choice B.

PASSAGE II

8.   The correct answer is J.  To answer this question, calculate the difference in diffusion time between each pair of gases in each answer choice:

F:  He and Kr: 13 and 43; the difference is 30 seconds

G:  Ne and Ar: 18 and 28; the difference is 10 seconds

H:  Kr and Rn: 43 and 73; the difference is 30 seconds

 J:  Ne and Xe: 18 and 58; the difference is 40 seconds

The greatest difference in diffusion time occurs between Ne and Xe, answer choice J.

9.   The correct answer is A.  According to Figure 1 and Table 1, as molecular mass (a.m.u.) increases, diffusion time also increases. The other answer choices are not supported by the data.

10.   The correct answer is F.  According to Table 1, He has the shortest diffusion time; it should completely diffuse first.

11.   The correct answer is C.  According to Table 1, the diffusion time of Ar is 28 seconds. Since 14 is half, or 50%, of 28, it is safe to assume that after 14 seconds, 50% of the vacuumed area will NOT have any Ar gas molecules left, answer choice C.

12.   The correct answer is J.  According to Table 1, in a 10-cubic-foot vacuumed area it takes Ne 18 seconds to diffuse. 18 x 3 is 54, so the volume of the vacuumed area described in the question is most likely 10 x 3, or 30 cubic feet, answer choice J.

PASSAGE III

13.   The correct answer is A.  Process A will be most accurate as compared to the Standard Method when the measurements obtained using Process A are similar to the measurements obtained using the Standard Method. If you look at Table 1 you see that, at a pH level of 2, the concentration of dissolved O2 is identical using both Process A and the Standard Method. Therefore, answer choice A is correct.

14.   The correct answer is H.  The first step in answering this question is to find calcium carbonate on Table 1. Then, see what happens to the concentration levels as you move across the table from left to right. In the Standard Method, Process A, and Process B, the concentration levels all increase significantly as the pH levels increase. Higher pH levels mean lower acidity.

15.   The correct answer is C.  Process B will be more accurate than Process A if the measurements obtained using Process B are similar to the measurements obtained using the Standard Method. Find NH3 on Table 1, and compare the results obtained from each method. You will see that, at each pH level, the results obtained using Process B are closer to the results obtained using the Standard Method. Therefore, you can eliminate answer choices A and B, which both say that Process A is more accurate. Answer choice C is most consistent with the data in Table 1, so it is correct.

16.   The correct answer is F.  First, find CO2 on Table 1, and look at the concentration levels obtained by using Process A. You will see that, from pH levels 2 through 5, there is a gradual reduction in CO2 concentration, but at a pH level of 6, the CO2 concentration jumps up dramatically. This is best represented by the graph in answer choice F.

17.   The correct answer is D.  To solve this problem, look at Table 1 and determine the pH level that corresponds the closest to the concentration values given in the problem for Process A. The data best supports answer choice D.

PASSAGE IV

18.   The correct answer is G.  To answer this question, look at Table 1 and Table 2 and determine which pesticide application resulted in the lowest number of fleas remaining. Note that the average number of fleas per dog before treatment in both experiments was 57. In both experiments, application of Pesticide B resulted in fewer fleas than did application of Pesticide A or Pesticide A+B. Therefore, answer choice G is correct.

19.   The correct answer is D.  According to Table 2, dogs with short coats had fewer ticks after all pesticide applications than did dogs with long coats. You can eliminate answer choices A and B. Now, look at Table 2 to determine whether Pesticide A or Pesticide B most reduced the number of ticks on a dog. Since all of the dogs started out with an average of 13 ticks before treatment, and the Pesticide A application resulted in only 4 ticks per dog, while the Pesticide B application resulted in 10 ticks per dog, answer choice D must be correct.

20.   The correct answer is H.  It is clear based on the data in Table 2 that shorter coat length leads to increased effectiveness of all pesticides, answer choice H.

21.   The correct answer is A.  The passage indicates that Pesticide A+B shampoo contained only 50% of each pesticide, while the other shampoos contained 100% of either Pesticide A or Pesticide B. Therefore, the most likely reason for the relative ineffectiveness of Pesticide A+B is that the two pesticides reduced each other’s effectiveness, answer choice A. The other answer choices are not supported by the passage.

22.   The correct answer is G.  Based on Table 3, an average coat length of 2 inches would be considered long. On Table 2, find the average number of ticks per dog 24 hours after application of Pesticide A. That number is 7, answer choice G.

PASSAGE V

23.   The correct answer is C.  The passage defines halflife as the “amount of time it takes for half of the atoms in a sample to decay.” Locate the line on the graph in Figure 1 that corresponds to Fluorine 22, and find the time at which half, or 50%, of the atoms are remaining in the sample. The half-life of Fluorine 22 is 4.2 seconds, answer choice C.

24.   The correct answer is H.  The passage indicates that radioactive decay is “a natural process by which an atom of a radioactive isotope spontaneously decays into another element.” The other answer choices are not supported by details in the passage.

25.   The correct answer is A.  Table 1 indicates that lower decay energy values result in lower particle velocity, answer choice A.

26.   The correct answer is J.  The decay energy of Cerium 53 into Lanthanum 127 (6.100) is closest to the decay energy of Oxygen 22 into Fluorine 22 (6.490). Therefore, it is likely that the particle velocity will be similar to Oxygen 22 as well. This best supports answer choice J.

27.   The correct answer is B.  Oxygen 22 and Neon 22 will have the same percent of atoms remaining at the point where the lines representing each product cross on the graph. When you locate the appropriate lines, you see that they cross at 4.5 seconds, answer choice B.

28.   The correct answer is H.  Both of the curves mentioned in the question show a rather rapid initial decay rate that appears to slow and stabilize as the percentage of atoms remaining is reduced:

Image

This best supports answer choice H.

PASSAGE VI

29.   The correct answer is B.  Table 1 shows the average number of eggs laid and the average number of eggs hatched before pesticide exposure. Since Breed F has a low resistance to illness, it would probably have a similar eggs hatched to eggs laid ratio as that of Breed E. Since 9 out of 15, or 3 out of 5 of Breed E’s eggs hatched, you can assume that the same fraction of Breed F’s eggs would hatch: Image answer choice B.

30.   The correct answer is H.  To answer this question, look at Figure 1 and find the breed that, at the far right end of the figure, has an illness incidence closest to zero. This will be Breed B, answer choice H.

31.   The correct answer is A.  According to Figure 1, both Breed E and Breed C have a high incidence of illness, so you can eliminate answer choices C and D. Although, according to Figure 2, Breed B lays fewer eggs than does Breed A, more of them hatch. Therefore, Breed B is likely to have a higher number of eggs hatch and have a low incidence of illness.

32.   The correct answer is G.  According to Table 1, the average number of eggs laid does not seem to be affected by the resistance to illness. Therefore, there is no direct relationship, answer choice G.

33.   The correct answer is D.  The breed that is least affected by pesticide exposure will likely be the breed that has the most consistent illness incidence both before and after exposure. According to Figure 1, Breed E had a high level of illness before pesticide exposure; illness incidences only increased slightly.

34.   The correct answer is J.  The only statement that is supported by the passage is that the average number of eggs laid by all breeds is not affected by pesticide exposure. The data in both Table 1 and Figure 2 supports this conclusion.

35.   The correct answer is A.  According to Table 1, Breed E laid an average of 15 eggs before exposure to pesticide, more than any other breed.

PASSAGE VII

36.   The correct answer is H.  Based on the data in all 3 tables, the highest average speed was recorded in Table 1Table 1 shows the results of Study 1, which placed the car on a smooth asphalt road. Therefore, the highest average speeds resulted from using an asphalt road, answer choice H.

37.   The correct answer is C.  The average speed recorded in Table 1 is 8.58 feet per second. This speed is not greater than the speed recorded in Trial 2 (8.85 ft/s); likewise, it is not less than the speed recorded in Trial 3 (8.55 ft/s); eliminate answer choices A and B. The average speed recorded in Table 1 (8.58 ft/s) is greater than the speed recorded in Trial 1 (8.33 ft/s), so answer choice C must be correct.

38.   The correct answer is F.  To answer this question, you must remember that Table 1 is associated with an asphalt road, Table 2 is associated with a gravel road, and Table 3 is associated with a dirt road. When you compare the average recorded speed, you will see that the average speed of a car on a gravel road (4.52 ft/s) is approximately half of the average speed of a car on an asphalt road (8.58 ft/s), answer choice F.

39.   The correct answer is B.  Since the passage indicates that all 3 of the studies were “conducted on a fair day with no wind,” you can eliminate answer choices C and D. The studies were also conducted over different ground cover, so the most likely reason for the lower average speeds is greater friction, answer choice B.

40.   The correct answer is H.  Look at each of the answer choices and compare the travel times:

F:  Study 1, Trial 2 = 8.85 ft/s

G:  Study 2, Trial 2 = 4.59 ft/s

H:  Study 2, Trial 3 = 4.46 ft/s

 J:  Study 3, Trial 3 = 6.17 ft/s

The slowest travel time was recorded in Study 2, Trial 3, answer choice H.